Fluids and Electrolytes Flashcards

1
Q

Metabolic acidosis with a normal anion gap occurs with

A. Diabetic acidosis

B. Renal failure

C. Severe diarrhea

D. Starvation

A

C

Metabolic acidosis with a normal anion gap (AG) results from either acid administration (HCl or NH4+) or a loss of bicarbonate from GI losses, such as diarrhea, fistulas (enteric, pancreatic, or biliary), ureterosigmoidosctomy, or from renal loss.

The bicarbonate loss is accompanied by a gain of chloride, thus the AG remains unchanged.

How well did you know this?
1
Not at all
2
3
4
5
Perfectly
2
Q

All are possible causes of postoperative hyponatremia
EXCEPT

A. Excess infusion of normal saline intraoperatively.

B. Administration of antipsychotic medication.

C. Transient decrease in antidiuretic hormone (ADH)
secretion.

D. Excess oral water intake.

A

C

Hyponatremia is caused by excess free water (dilution) or decreased sodium (depletion). Thus, excessive intake of free water (oral or IV) can lead to hyponatremia. Also, medications can cause water retention and subsequent hyponatremia, especially in older patients.

Primary renal disease, diuretic use, and secretion of antidiuretic hormone (ADH) are common causes of sodium depletion.

ADH can be released transiently postoperatively, or less frequently, in syndrome of inappropriate ADH secretion.

Lastly, pseudohyponatremia can be seen on laboratory testing when high serum glucose, lipid, or protein levels compromise sodium measurements.

(See Schwartz 10th ed., p. 69.)

How well did you know this?
1
Not at all
2
3
4
5
Perfectly
3
Q

Which of the following is an early sign of hyperkalemia?

A. Peaked T waves

B. Peaked P waves

C. Peaked (shortened) QRS complex

D. Peaked U waves

A

Symptoms of hyperkalemia are primarily GI, neuromuscular, and cardiovascular. GI symptoms include nausea, vomiting, intestinal colic, and diarrhea; neuromuscular symptoms range from weakness to ascending paralysis to respiratory failure; while cardiovascular manifestations range from electrocardiogram (ECG) changes to cardiac arrhythmias and arrest.

ECG changes that may be seen with hyperkalemia include:

1) Peaked T waves (early change)
2) Flattened P wave
3) Prolonged PR interval (first-degree block)
4) Widened QRS complex
5) Sine wave formation
6) Ventricular fibrillation

How well did you know this?
1
Not at all
2
3
4
5
Perfectly
4
Q

Hypocalcemia may cause which of the following?

A. Congestive heart failure

B. Atrial fibrillation

C. Pancreatitis

D. Hypoparathyroidism

A

Mild hypocalcemia can present with muscle cramping or digital/perioral paresthesias.

Severe hypocalcemia leads to decreased cardiac contractility and heart failure.

ECG changes of hypocalcemia include prolonged QT interval, T-wave inversion, heart block, and ventricular fibrillation.

Hypoparathyroidism and severe pancreatitis are potential causes of hypocalcemia.

(See Schwartz 10th ed., p. 72.)

How well did you know this?
1
Not at all
2
3
4
5
Perfectly
5
Q

The next most appropriate test to order in a patient with a pH of 7.1, Pco2 of 40, sodium of 132, potassium of 4.2, and chloride of 105 is

A. Serum bicarbonate

B. Serum magnesium

C. Serum ethanol

D. Serum salicylate

A

A

Metabolic acidosis results from an increased intake of acids, an increased generation of acids, or an increased loss of bicarbonate. In evaluating a patient with a low serum bicarbonate level and metabolic acidosis, first measure the AG, an index of unmeasured anions.

AG= [Na] - [Cl + HCO3]

Metabolic acidosis with an increased AG occurs from either exogenous acid ingestion (ethylene glycol, salicylate, or methanol) or endogenous acid production of ß-hydroxybutyrate and acetoacetate in ketoacidosis, lactate in lactic acidosis, or organic acids in renal insufficiency. (See Schwartz 10th ed., p. 74.)

How well did you know this?
1
Not at all
2
3
4
5
Perfectly
6
Q

Which of the following is FALSE regarding hypertonic
saline?

A. Is an arteriolar vasodilator and may increase bleeding

B. Should be avoided in closed head injury

C. Should not be used for initial resuscitation

D. Increases cerebral perfusion

A

B

Hypertonic saline (7.5%) has been used as a treatment modality in patients with closed head injuries. It has been shown to increase cerebral perfusion and decrease intracranial pressure, thus decreasing brain edema.

However, there also have been concerns of increased bleeding because hypertonic saline is an arteriolar vasodilator.

(See Schwartz 10th ed., p. 76.)

How well did you know this?
1
Not at all
2
3
4
5
Perfectly
7
Q

Normal saline is

A. 135mEqNaCl/L

B. 145mEqNaCVL

C. 148mEqNaCl/L

D. 154mEqNaCl/L

A

D

Sodium chloride is mildly hypertonic, containing 154 mEq of sodium that is balanced by 154 mEq of chloride.

The high chloride concentration imposes a significant chloride load upon the kidneys and may lead to a hyperchloremic metabolic acidosis.

It is an ideal solution, however, for correcting volume deficits associated with hyponatremia, hypochloremia, and metabolic alkalosis.

(See Schwartz 10th ed., p. 76.)

How well did you know this?
1
Not at all
2
3
4
5
Perfectly
8
Q

Fluid resuscitation using albumin

A. Is associated with coagulopathy

B. Is available as 1% or 5% solutions

C. Can lead to pulmonary edema

D. Decreased factor XIII

A

C

Albumin is available as 5% (osmolality of 300 mOsm/L) or 25% (osmolality of 1500 mOsm/L).

Due to increased intravascular oncotic pressure, fluid is drawn into the intravascular space, leading to pulmonary edema when albumin is used for resuscitation for hypovolemic shock.

Hydroxyethyl starch solutions are associated with postoperative bleeding in cardiac and neurosurgery patients.

(See Schwartz 10th ed., p. 77.)

How well did you know this?
1
Not at all
2
3
4
5
Perfectly
9
Q

Water constitutes what percentage of total body weight?

A. 30-40%

B. 40-50%

C. 50-60%

D. 60-70%

A

C

Water constitutes approximately 50 to 60% of total body weight. The relationship between total body weight and total body water (TBW) is relatively constant for an individual and is primarily a reflection of body fat.

Lean tissues, such as muscle and solid organs, have higher water content than fat and bone. As a result, young, lean men have a higher proportion of body weight as water than elderly or obese individuals.

An average young adult male will have 60% of his total body weight as TBW, while an average young adult female’s will be 50%. The lower percentage of TBW in women correlates with a higher percentage of adipose tissue and lower percentage of
muscle mass in most.

Estimates of TBW should be adjusted down approximately 10 to 20% in obese individuals and up by 10% in malnourished individuals. The highest percentage of TBW is found in newborns, with approximately 80% of their total bodyweight composed of water.

This decreases to about 65% by 1 year and thereafter remains fairly constant.

(See Schwartz 10th ed., p. 65.)

How well did you know this?
1
Not at all
2
3
4
5
Perfectly
10
Q

If a patient’s serum glucose increases by 180 mg/ dL, what is the increase in serum osmolality, assuming all other laboratory values remain constant?

A. Does not change

B. 8

C. 10

D. 12

A

Answer: C

Osmotic pressure is measured in units of osmoles (osm) or milliosmoles (mOsm) that refer to the actual number of osmotically active particles. For example, 1 millimole (mmol) of sodium chloride contributes to 2 mOsm (one from sodium and one from chloride).

The principal determinants of osmolality are the concentrations of sodium, glucose, and urea (blood urea nitrogen [BUN]):

Calculated serum osmolality = 2sodium + glucose/18 + BUN/2.8

(See Schwartz 10th ed., p. 67.)

How well did you know this?
1
Not at all
2
3
4
5
Perfectly
11
Q

What is the actual potassium of a patient with pH of 7.8
and serum potassium of 2.2?

A. 2.2

B. 2.8

C. 3.2

D. 3.4

A

Answer: D

The change in potassium associated with alkalosis can be calculated by the following formula:

Potassium decreases by 0.3 mEq/Lfor every 0.1 increase in pH above normal.

(See Schwartz 10th ed., p. 71.)

How well did you know this?
1
Not at all
2
3
4
5
Perfectly
12
Q

The free water deficit of a 70 kg man with serum sodium of 154 is

A. 0.1 L

B. 0.7 L

C. IL

D. 7L

A

D

This is the formula used to estimate the amount of water required to correct hypernatremia:

Water deficit L = (serum sodium - 140)/140 x TBW

Estimate TBW (total body water) as 50% of lean body mass in men and 40% in women.

(See Schwartz 10th ed., p. 69.)

How well did you know this?
1
Not at all
2
3
4
5
Perfectly
13
Q

A patient with serum calcium of 6.8 and albumin of 1.2 has a corrected calcium of

A. 7.7

B. 8.0

C. 8.6

D. 9.2

A

Answer: D

When measuring total serum calcium levels, the albumin concentration must be taken into consideration.

Adjust total serum calcium down by 0.8 mg/dL for every 1 g/dL decrease in albumin.

(See Schwartz 10th ed., p. 72.)

How well did you know this?
1
Not at all
2
3
4
5
Perfectly
14
Q

All the following treatments for hyperkalemia reduce serum potassium EXCEPT:

A. Bicarbonate

B. Kayexalate

C. Glucose infusion with insulin

D. Calcium

A

Answer: D

When ECG changes are present, calcium chloride or calcium gluconate (5-10 mL of 10% solution) should be administered immediately to counteract the myocardial effects of hyperkalemia.

Calcium infusion should be used cautiously in patients receiving digitalis, because digitalis toxicity may be precipitated.

Glucose and bicarbonate shift potassium intracellularly. Kayexalate is a cation exchange resin that binds potassium, either given enterally or as an enema.

(See Schwarz 10th ed., p. 77.)

How well did you know this?
1
Not at all
2
3
4
5
Perfectly
15
Q

An alcoholic patient with serum albumin of 3.9, K of 3.1, Mg of 2.4, Ca of 7.8, and PO4 of 3.2 receives three boluses of IV potassium and has serum potassium of 3.3. You should:

A. Continue to bolus potassium until the serum level is >3.6.

B. Give MgSO4 IV.

C. Check the ionized calcium.

D. Check the BUN and creatinine.

A

Answer: B

Magnesium depletion is a common problem in hospitalized patients, particularly in the ICU. The kidney is primarily responsible for magnesium homeostasis through regulation by calcium/magnesium receptors on renal tubular cells that sense serum magnesium levels.

Hypomagnesemia results from a variety of etiologies ranging from poor intake (starvation, alcoholism, prolonged use of IV fluids, and total parenteral nutrition with inadequate supplementation of magnesium), increased renal excretion (alcohol, most diuretics, and amphotericin B), GI losses (diarrhea), malabsorption, acute pancreatitis, diabetic ketoacidosis, and primary aldosteronism.

Hypomagnesemia is important not only for its direct effects on the nervous system but also because it can produce hypocalcemia and lead to persistent hypokalemia. When hypokalemia or hypocalcemia coexist with hypomagnesemia, magnesium should be aggressively replaced to assist in restoring potassium or calcium homeostasis.

(See Schwartz 10th ed., p. 73.)

How well did you know this?
1
Not at all
2
3
4
5
Perfectly
16
Q

Calculate the daily maintenance fluids needed for a 60-kg female:

A. 2060

B. 2100

C. 2160

D. 2400

A

Answer: B

A 60-kg female would receive a total of 2100 mL of fluid daily:

1000 mL for the first 10 kg of body weight ( 10 kg x 100 mL/kg/day),
500 mL for the next 20 kg (10 kg x 50 mL/kg/day), and
80 mL for the last 40 kg (40 kg x 20 mL/kg/day).

How well did you know this?
1
Not at all
2
3
4
5
Perfectly
17
Q

A patient who has spasms in the hand when a blood pressure cuff is blown up most likely has

A. Hypercalcemia

B. Hypocalcemia

C. Hypermagnesemia

D. Hypomagnesemia

A

Answer: B

Asymptomatic hypocalcemia may occur with hypoproteinemia (normal ionized calcium), but symptoms can develop with alkalosis (decreased ionized calcium).

In general, symptoms do not occur until the ionized fraction falls below 2.5 mg/dL, and are neuromuscular and cardiac in origin, including paresthesias of the face and extremities, muscle cramps, carpopedal spasm, stridor, tetany, and seizures.

Patients will demonstrate hyperreflexia and positive Chvostek sign (spasm resulting from tapping over the facial nerve) and Trousseau sign (spasm resulting from pressure applied to the nerves and vessels of the upper extremity, as when obtaining a blood pressure).

Decreased cardiac contractility and heart failure can also accompany hypocalcemia.

(See Schwartz 10th ed., p. 72.)

How well did you know this?
1
Not at all
2
3
4
5
Perfectly
18
Q

The actual AG of a chronic alcoholic with Na 133, K 4, Cl 101, HCO3 22, albumin of 2.5 mg/dL is:

A. 6

B. 10

C. 14

D. 15

A

Answer: D

The normal AG is <12 mmol/L and is due primarily to the albumin effect, so that the estimated AG must be adjusted for albumin (hypoalbuminemia reduces the AG).

Corrected AG = actual AG + [2.5(4.5 - albumin)]

(See Schwartz 10th ed., p. 74.)

How well did you know this?
1
Not at all
2
3
4
5
Perfectly
19
Q

The effective osmotic pressure between the plasma and interstitial fluid compartments is primarily controlled by

A. Bicarbonate

B. Chloride ion

C. Potassium ion

D. Protein

A

Answer: D

The dissolved protein in plasma does not pass through the semipermeable cell membrane, and this fact is responsible for the effective or colloid osmotic pressure.

(See Schwartz 10th ed.,p. 66.)

How well did you know this?
1
Not at all
2
3
4
5
Perfectly
20
Q

The metabolic derangement most commonly seen in
patients with profuse vomiting

A. Hypochloremic, hypokalemic metabolic alkalosis

B. Hypochloremic, hypokalemic metabolic acidosis

C. Hypochloremic, hyperkalemic metabolic alkalosis

D. Hypochloremic, hyperkalemic metabolic acidosis

A

A

Hypochloremic, hypokalemic metabolic alkalosis can occur from isolated loss of gastric contents in infants with pyloric stenosis or in adults with duodenal ulcer disease.

Unlike vomiting associated with an open pylorus, which involves a loss of gastric as well as pancreatic, biliary, and intestinal secretions, vomiting with an obstructed pylorus results only in the loss of gastric fluid, which is high in chloride and hydrogen, and therefore results in a hypochloremic alkalosis.

Initially the urinary bicarbonate level is high in compensation for the alkalosis. Hydrogen ion reabsorption also ensues, with an accompanied potassium ion excretion. In response to the associated
volume deficit, aldosterone-mediated sodium reabsorption increases potassium excretion.

The resulting hypokalemia leads to the excretion of hydrogen ions in the face of alkalosis, a paradoxic aciduria.

Treatment includes replacement of the volume deficit with isotonic saline and then potassium replacement once adequate urine output is achieved.

(See Schwartz 10th ed., p. 74.)

How well did you know this?
1
Not at all
2
3
4
5
Perfectly
21
Q

Symptoms and signs of extracellular fluid volume deficit include all of the following, EXCEPT:

A. Anorexia

B. Apathy

C. Decreased body temperature

D. High pulse pressure

A

Answer: D

High pulse pressure occurs with extracellular fluid volume excess, but the other symptoms and signs are characteristic of moderate extracellular volume deficit.

(See Schwartz 10th ed., p.68)

How well did you know this?
1
Not at all
2
3
4
5
Perfectly
22
Q

A low urinary [NH4+] with a hyperchloremic acidosis
indicates what cause?

A. Excessive vomiting

B. Enterocutaneous fistula

C. Chronic diarrhea

D. Renal tubular acidosis

A

Answer: D

Metabolic acidosis with a normal AG results either from
exogenous acid administration (HC1 or NH4+), from loss of bicarbonate due to GI disorders such as diarrhea and fistulas or ureterosigmoidostomy, or from renal losses.

In these settings, the bicarbonate loss is accompanied by a gain of chloride; thus, the AG remains unchanged.

To determine if the loss of bicarbonate has a renal cause, the urinary [NH4+] can be measured.

A low urinary [NH4+] in the face of hyperchloremic acidosis would indicate that the kidney is the site of loss, and evaluation for renal tubular acidosis should be undertaken.

Proximal renal tubular acidosis results from decreased
tubular reabsorption of HCO3 , whereas distal renal tubular acidosis results from decreased acid excretion.

The carbonic anhydrase inhibitor acetazolamide also causes bicarbonate loss from the kidneys.

(See Schwartz 10th ed., p. 74.)

How well did you know this?
1
Not at all
2
3
4
5
Perfectly
23
Q

When lactic acid is produced in response to injury, the
body minimizes pH change by

A. Decreasing production of sodium bicarbonate in
tissues

B. Excreting carbon dioxide through the lungs

C. Excreting lactic acid through the kidneys

D. Metabolizing the lactic acid in the liver

A

Answer: B

Lactic acid reacts with base bicarbonate to produce carbonic acid.

The carbonic acid is broken down into water and carbon dioxide that is excreted by the lungs.

Any diminution in pulmonary function jeopardizes this reaction.

(See Schwartz 10th ed.,p. 73.)

How well did you know this?
1
Not at all
2
3
4
5
Perfectly
24
Q

What is the best determinant of whether a patient has a
metabolic acidosis versus alkalosis?

A. Arterial pH

B. Serum bicarbonate

C. Pco2

D. Serum CO, level

A

Answer: A

While bicarbonate, Pco2, and patient history often can suggest the most likely metabolic derangement, only the measurement of arterial pH confirms acidosis versus alkalosis.

(See Schwartz 10th ed., p. 74.)

How well did you know this?
1
Not at all
2
3
4
5
Perfectly
25
Q

If a patient’s arterial Pco2 is found to be 25 mm Hg, the
arterial pH will be approximately

A. 7.52

B. 7.40

C. 7.32

D. 7.28

A

Answer: D

A low PaCO2 indicates excess elimination of carbon dioxide by the lungs, and the body pH will fall.

Within reasonable physiologic ranges a 15 mm Hg fall in Paco, should produce a 0.12 change from the normal body pH of 7.4.

(See Schwartz 10th ed.,p.74.)

How well did you know this?
1
Not at all
2
3
4
5
Perfectly
26
Q

Which of the following are NOT characteristic findings of acute renal failure?

A. BUN>100mg/dL

B. Hypokalemia

C. Severe acidosis

D. Uremic pericarditis

E. Uremic encephalopathy

A

Answer: A

Hyperkalemia, severe acidosis, uremic encephalopathy, and uremic pericarditis are all indications of life-threatening problems, and urgent correction is mandatory.

Elevation of BUN is commonly seen as well, but is not itself an indication for dialysis.

(See Schwartz 10th ed., p. 81.)

How well did you know this?
1
Not at all
2
3
4
5
Perfectly
27
Q

An elderly diabetic patient who has acute cholecystitis is found to have a serum sodium level of 122 mEq/L and a blood glucose of 600 mg/dL. Af er correcting the glucose concentration to 100 mg/dL with insulin, the serum sodium concentration would

A. Decrease significantly unless the patient also received 3% saline

B. Decrease transiently but return to approximately 122 mEq/L without specific therapy

C. Remain essentially unchanged

D. Increase to the normal range without specific therapy

A

Answer: D

A rise in the extracellular fluid concentration of a substance that does not diffuse passively across cell membranes (eg, glucose or urea) causes an increase in effective osmotic pressure, a transfer of water from cells, and dilutional hyponatremia.

For each 100 mg/dLrise in blood glucose above normal, the serum sodium level falls approximately to 3 mEq/L. Alternatively, the serum sodium level would increase by about 15 mEq/L if the blood glucose level fell from 600 to 100mg/dL.

(See Schwartz 10th ed., p. 69.)

How well did you know this?
1
Not at all
2
3
4
5
Perfectly
28
Q

Excessive administration of normal saline for fluid resuscitation can lead to what metabolic derangement?

A. Metabolic alkalosis

B. Metabolic acidosis

C. Respiratory alkalosis

D. Respiratory acidosis

A

Answer: B

Sodium chloride is mildly hypertonic, containing 154 mEq of sodium that is balanced by 154 mEq of chloride, The high chloride concentration imposes a significant chloride load on the kidneys and may lead to a hyperchloremic metabolic acidosis.

Sodium chloride is an ideal solution, however, for correcting volume deficits associated with hyponatremia, hypochloremia, and metabolic alkalosis.

(See Schwartz 10th ed., p. 74.)

How well did you know this?
1
Not at all
2
3
4
5
Perfectly
29
Q

The first step in the management of acute hypercalcemia
should be

A. Correction of deficit of extracellular fluid volume

B. Hemodialysis.

C. Administration of furosemide.

D. Administration of mithramycin.

A

Answer: A

Patients with acute hypercalcemia usually have either acute hyperparathyroidism or metastatic breast carcinoma with multiple bony metastases. These patients develop severe headaches, bone pain, thirst, emesis, and polyuria.

Unless treatment is instituted promptly, the symptoms may be rapidly fatal. Immediate correction of the associated deficit of extracellular fluid volume is the most important step in treatment.

When effective, this results in the lowering of the serum
calcium level by dilution. Once extracellular fluid volume has been replaced, furosemide is effective treatment.

Hemodialysis may also be employed, but its effect is less rapid.

Mithramycin is very useful in controlling metastatic bone disease, but its effect is slow, and it cannot be depended upon when the patient has acute hypercalcemia.

(See Schwartz 10th ed., p. 72.)

How well did you know this?
1
Not at all
2
3
4
5
Perfectly
30
Q

A victim of a motor vehicle accident arrives in hemorrhagic shock. His arterial blood gases are pH, 7.25; Po2, 95 mm Hg; Pco2, 25 mm Hg; HCO3~, 15 mEq/L. The patient’s metabolic acidosis would be treated best with:

A. Ampule of sodium bicarbonate

B. Sodium bicarbonate infusion

C. Lactated Ringer solution

D. Hyperventilation

A

Answer: C

In patients suffering from hemorrhagic shock, the presence of a metabolic acidosis early in the postresuscitative period is indicative of tissue hypoxia due to persistent inadequate tissue perfusion.

Attempts to correct this problem by administering an alkalizing agent will not solve the basic problem.

How well did you know this?
1
Not at all
2
3
4
5
Perfectly
31
Q

Three days after surgery for gastric carcinoma, a 50-year-old alcoholic male exhibits delirium, muscle tremors, and hyperactive tendon reflexes. Magnesium deficiency is suspected. All of the following statements regarding this situation are true EXCEPT

A. A decision to administer magnesium should be based
on the serum magnesium level.

B. Adequate cellular replacement of magnesium will
require 1 to 3 weeks.

C. A concomitant calcium deficiency should be suspected.

D. Calcium is a specific antagonist of the myocardial
effects of magnesium.

A

Answer: A

Magnesium deficiency should be suspected in any malnourished patient who exhibits disturbed neuromuscular or cerebral activity in the postoperative period.

Laboratory confirmation often is not reliable, and the syndrome may exist in the presence of a normal serum magnesium level.

Hypocalcemia often coexists, particularly in patients who have clinical signs of tetany. Intravenous magnesium can be administered safely to a well-hydrated patient for initial treatment of a severe deficit, but concomitant electrocardiographic monitoring is essential.

The electrocardiographic changes associated with
acute hypermagnesemia resemble those of hyperkalemia, and calcium chloride or gluconate should be readily available to counteract any adverse myocardial effects of excess magnesium ions.

Partial or complete relief of symptoms may follow the initial infusion of magnesium, although continued replacement for a period of 1 to 3 weeks is necessary to replenish cellular stores.

(See Schwartz 10th ed., p. 78.)

How well did you know this?
1
Not at all
2
3
4
5
Perfectly
32
Q

Refeeding syndrome can be associated with all of the
following EXCEPT

A. Respiratoryfailure

B. Hyperkalemia

C. Confusion

D. Cardiac arrhythmias I

A

Answer: B

With refeeding, a shift in metabolism from fat to carbohydrate substrate stimulates insulin release, which results in the cellular uptake of electrolytes, particularly phosphate, magnesium, potassium, and calcium.

However, severe hyperglycemia may result from blunted basal insulin secretion.

How well did you know this?
1
Not at all
2
3
4
5
Perfectly
33
Q

Which one of the following clinical scenarios is associated with hypercalcemia?

A. Fluid resuscitation from shock

B. Rapid infusion of blood products

C. Improper administration of phosphates

D. Malignancy

E. Acute pancreatitis

A

ANSWER: D

COMMENTS: Infusion of large volumes of isotonic fluid can cause a modest reduction in serum calcium levels.

The concomitant decrease in magnesium also impairs vitamin D activity and makes correction of the hypocalcemia more difficult.

Administration of a citrate load during rapid transfusion of blood products can lead to severe hypocalcemia, hypotension, and cardiac failure.

In this setting, calcium should be replaced at a dose of 0.2 g/500 mL of blood transfused. Most patients receiving blood transfusions do not require calcium supplementation.

Acute pancreatitis causes precipitation of calcium salts in the abdomen and may contribute to hypocalcemia.

Other common causes include necrotizing fasciitis, renal failure, gastrointestinal fistula, and hypoparathyroidism. In general, calcium replacement should be monitored by measuring the concentration of ionized calcium.

How well did you know this?
1
Not at all
2
3
4
5
Perfectly
34
Q

A 30-year-old, 70-kg woman has symptomatic hyponatremia. Her serum sodium level is 120 mEq/L (normal level, 140 mEq/L). Her sodium deficit is:

A. 500 mEq/L

B. 600 mEq/L

C. 700 mEq/L

D. 800 mEq/L

E. 400 mEq/L

A

ANSWER: C

COMMENTS: Correction of changes in concentration depends, in part, on whether the patient is symptomatic. If symptomatic hypernatremia or hyponatremia is present, attention is focused on prompt correction of the abnormal concentration to the point that the symptoms are relieved. Attention is then shifted to correction of the associated abnormality in volume.

The sodium deficiency in such patients is estimated by multiplying the sodium deficit (normal sodium concentration minus observed sodium concentration) by total body water in liters (60% of body weight in males and 50% of body weight in females).

For the patient in question, the calculation is as follows:
total body water = 70 kg × 0.5 = 35 L.

Sodium deficit = (140 − 120 mEq/L) × 35 L = 700 mEq sodium chloride.

Initially, half the calculated amount of sodium is infused as 3% sodium chloride. The infusion is given slowly because rapid infusion can cause symptomatic hypovolemia.

Rapid correction of hyponatremia can be associated with irreversible central nervous system injury (central pontine and extrapontine myelinolysis).

Once the symptoms are alleviated, the patient should be reassessed before beginning the additional infusion of sodium. In patients with profound hyponatremia, a correction of no more than 12 mEq/L/24 h should be achieved.

If the original problem was associated with a volume deficit, the remainder of the resuscitation can be accomplished with isotonic fluids (sodium chloride in the presence of alkalosis and sodium lactate in the presence of acidosis).

Care must be taken when treating hyponatremia associ- ated with volume excess. In this setting, after the symptoms are alleviated with a small volume of hypertonic saline solution, water restriction is the treatment of choice.

Infusion of hypertonic saline solution in this setting has the potential to further expand the extracellular intravascular volume and is contraindicated in patients with severely compromised cardiac reserve. In such a case, peritoneal dialysis or hemodialysis may be preferred for removing excess water.

How well did you know this?
1
Not at all
2
3
4
5
Perfectly
35
Q

Which of the following disturbances is associated with tumor lysis syndrome?

A. Hypocalcemia

B. Hypouricemia

C. Hypokalemia

D. Hypomagnesemia

E. Hypophosphatemia

A

ANSWER: A

COMMENTS: Tumor lysis syndrome is a constellation of electrolyte abnormalities that results from massive tumor cell necrosis secondary to antineoplastic therapy.

Hypocalcemia, hyperphos- phatemia, hyperuricemia, and hyperkalemia may occur.

Hypocalcemia results from the release of intracellular stores of phosphate, which binds with ionized serum calcium to form calcium phosphate salts.

Chemotherapy directed against solid tumors, especially lymphomas, is most commonly associated with tumor lysis syndrome.

Acute renal failure can occur and prevent spontaneous correction of the electrolyte abnormalities.

Hypermagnesemia is not associated with tumor lysis syndrome.

How well did you know this?
1
Not at all
2
3
4
5
Perfectly
36
Q

An elderly patient with adult-onset diabetes mellitus is admitted to the hospital with severe pneumonia. The patient would not be expected to develop:

A. Hypokalemia

B. Hyperkalemia

C. Nonketotic hyperosmolar coma

D. Hypophosphatemia

E. Hyponatremia

A

ANSWER: B

COMMENTS: Elderly patients with adult-onset diabetes mellitus are at risk for the development of nonketotic hyperosmolar coma during sepsis.

As a result of the development of a nonketotic hyperglycemic hyperosmolar state, hypokalemia and hyperglycemia may also occur.

Treatment of these patients should include a reduction in the glucose load provided and the administration of isotonic fluids.

Patients may also benefit from the administration of insulin.

Systemic bacterial sepsis is also often accompanied by a drop in the serum sodium con- centration, possibly because of interstitial or intracellular sequestration.

It is treated by withholding free water, restoring extracellular fluid (ECF) volume, and treating the source of sepsis.

How well did you know this?
1
Not at all
2
3
4
5
Perfectly
37
Q

An asymptomatic patient is found to have a serum calcium level of 13.5 mg/dL. Which of the following medications should be avoided?

A. Bisphosphonates

B. Thiazide diuretics

C. Mithramycin

D. Calcitonin

E. Corticosteroids

A

ANSWER: B

Hypercalcemia can affect the gastrointestinal, renal, musculoskeletal, and central nervous systems.

Early symptoms include fatigability, lassitude, weakness, anorexia, nausea, and vomiting.

Central nervous symptoms can progress to stupor and coma.

Other symptoms include headaches and the three Ps: pain, polydipsia, and polyuria.

The critical serum calcium level for hypercalcemia is 16 to 20 mg/mL. Prompt treatment must be instituted at this level, or the symptoms may progress to death.

Two major causes of hypercalcemia are hyperparathyroidism and metastatic disease.

Metastatic breast cancer in patients receiving estrogen therapy is the most common cause of hypercalcemia asso- ciated with metastases.

Oral or intravenous phosphates are useful for reducing hypercalcemia by inhibiting bone resorption and forming calcium phosphate complexes that are deposited in the soft tissues.

Intravenous phosphorus, however, has been associated with the acute development of hypocalcemia, hypotension, and renal failure. For this reason, it should be given slowly over a period of 8 to 12 h once daily for no more than 2 to 3 days.

Intravenous sodium sulfate is effective but no more so than saline diuresis.

Bisphosphonates reduce serum calcium levels by suppressing the function of osteoclasts and thus reducing the bone resorption of calcium.

In some malignant conditions, such as breast cancer, bisphosphonates may be administered prophylactically to prevent hypercalcemia.

Mithramycin lowers serum calcium levels within 24 to 48h by inhibiting bone resorption. A single dose may normalize serum calcium levels for several weeks.

Calcitonin is produced by the parafollicular cells of the thyroid gland and functions by inducing renal excretion of calcium and suppressing osteoclast bone resorption. Calcitonin can produce a moderate decrease in serum sodium levels, but the effect is lost with repeated administration.

Because corticosteroids decrease resorption of calcium from bone and reduce intestinal absorption, they are useful for treating hypercalcemic patients with sarcoidosis, myeloma, lymphoma, or leukemia. Their effects, however, may not be apparent for 1 to 2 weeks.

Chelating agents, such as ethylene- diaminetetraacetic acid (EDTA), are not indicated since they can result in metastatic calcification, acute renal failure, and hypocalcemia.

Thiazide diuretics are contraindicated because they are calcium sparing (and are often implicated as a cause of iatrogenic hypercalcemia).

Acute hypercalcemic crisis from hyperparathyroidism is treated by stabilizing the patient and performing a parathyroidectomy.

How well did you know this?
1
Not at all
2
3
4
5
Perfectly
38
Q

Which of the following statements regarding the distribution, composition, and osmolarity of body fluid compartments is true?

A. A majority of intracellular water resides in adipose tissue.

B. The principal intracellular anions are proteins and phosphates.

C. Sodium determines the effective osmotic pressure between the interstitial and intravascular (plasma) fluid compartments.

D. Calcium greatly determines the effective osmotic pressure between the intracellular fluid (ICF) and ECF compartments.

E. The principal intracellular cation is calcium.

A

ANSWER: B

The ICF compartment (accounting for 40% of total body weight) is contained mostly within skeletal muscles. The principal intracellular cations are potassium and magnesium, whereas the principal intracellular anions are proteins and phosphates. In the ECF compartment (20% of total body weight), which is subdivided into the interstitial (extravascular) and the intravascular (plasma) fluid compartments, the principal cation is sodium, whereas the principal anions are chloride and bicarbonate.

The interstitial compartment has a rapidly equilibrating functional component and a slowly equilibrating, relatively nonfunctional component consisting of fluid within connective tissue and cerebrospinal and joint fluid (termed transcellular water). Intravascular fluid (plasma) has a higher concentration of nondiffusible organic pro- teins than interstitial fluids. These plasma proteins act as multiva- lent anions.

As a result, the concentration of inorganic anions is lower, but the total concentration of cations is higher in the intra- vascular fluid than in the interstitial fluid.

This relationship is explained in the Gibbs–Donnan equilibrium equation: the product of the concentrations of any pair of diffusible cations and anions on one side of a semipermeable membrane equals the product of the same pair on the other side.
In each body compartment the concentration of osmotically active particles is 290 to 310 mOsm.

Although total osmotic pressure represents the sum of osmotically active particles in the fluid compartment, the effective osmotic pressure depends on osmotically active particles that do not freely pass through the semipermeable membranes of the body.

The nonpermeable proteins in plasma are responsible for the effective osmotic pressure between plasma and the interstitial fluid compartment (the colloid osmotic pressure).

The effective osmotic pressure between the ECF and ICF compartments is due mainly to sodium, the major extracellular cation, which does not freely cross the cell membrane. Because water moves freely between the compartments, the effective oncotic pressure within the various body fluid compartments is considered to be equal after fluid equilibration.

An increase in the effective oncotic pressure of the ECF compartment (such as an increase in sodium concentration) causes movement of water from the intracellular space to the extra- cellular space until the osmotic pressure equalizes.

Conversely, loss of sodium (hyponatremia) from the extracellular space results in movement of water into the intracellular space. Thus the ICF contributes to correcting the changes in concentration and composition in the ECF.

Isotonic ECF losses (losses in volume without change in concentration) generally do not cause transfer of water from the intracellular space as long as the osmolarity remains unchanged.

Isotonic volume losses result in changes in ECF volume.

How well did you know this?
1
Not at all
2
3
4
5
Perfectly
39
Q

Which of the following conditions is associated with
hypernatremia?

A. Adrenal insufficiency

B. Tumor lysis syndrome

C. Marked hyperglycemia

D. Stevens–Johnson syndrome

E. Excessive loop diuretic administration

A

ANSWER: D

Dermatologic conditions such as second-degree burns and exfoliative dermatitis can substantially increase transcutaneous water loss and thereby result in the rapid onset of dehydration and hypernatremia.

Tumor lysis syndrome, a condition involving cell breakdown and release of their intracellular contents after some chemotherapies, typically develops in patients treated with vinca alkaloid chemotherapy; it causes hyperkalemia, hyperphosphatemia, hyperuricemia, and ultimately, renal failure. Tumor lysis syndrome does not cause hypernatremia. Adrenal insuffi- ciency, hyperglycemia, and loop diuretics all cause hyponatremia.

How well did you know this?
1
Not at all
2
3
4
5
Perfectly
40
Q

Which of the following humoral factors increases arterial vasodilation while not decreasing protein permeability in the capillary membranes?

A. Bradykinin

B. Nitric oxide (NO)

C. Atrial natriuretic factor

D. Histamine

E. Platelet-activating factor

A

ANSWER: B

The protein permeability characteristics of capillary membranes are quantified by a numeric value termed the reflection coefficient. This value ranges from 0 to 1 and is conceptualized as the fraction of plasma protein that “reflects” back from the capillary wall when water crosses.

The higher the coefficient, the more impermeable the capillary is to protein.

Therefore the oncotic pressure of the plasma volume declines as the reflection coefficient decreases.

Certain intravascular factors can reduce the reflection coefficient and increase arterial vasodilation.

Bradykinin, atrial natriuretic factor, histamine, and platelet-activating factor increase microvascular membrane permeability while causing arterial vasodilation. NO, although it causes arterial vasodilation, does not increase microvascular membrane permeability. Membrane permeability causes a shift of fluid and plasma proteins into the interstitium and thereby decreases the intravascular compartment.

The protein-rich edema in the interstitium can adversely affect the ability to combat infection.

How well did you know this?
1
Not at all
2
3
4
5
Perfectly
41
Q

A 55-year-old female with a small bowel obstruction is found to have a serum potassium level of 2.8 mmol/L. Her hypokalemia is refractory to aggressive repletion. Which of the following is true?

A. The patient will likely suffer from flaccid paralysis and respiratory compromise until her potassium level is increased to at least 3.0 mmol/L.

B. An electrocardiogram will likely show peaked T waves.

C. Intravenous potassium repletion with a rate of 80 mEq/h should improve her condition.

D. Hypomagnesemia could contribute to her problem.

E. Hypokalemia results in hypopolarization of the resting
potential of the cell.

A

ANSWER: D

COMMENTS: Potassium is the main intracellular ion. Patients with a potassium concentration lower than 3.5 mmol/L have hypokalemia, which results in hyperpolarization of the resting potential of the cell and interferes with neuromuscular function.

In severe cases where the serum potassium falls below 2.0 mmol/L, flaccid paralysis with respiratory compromise can occur.

In the setting of hypokalemia, an electrocardiogram may show depressed T waves and U waves. Cardiac arrhythmia, particularly atrial tachycardia with or without a block; atrioventricular dissociation; ventricular tachycardia; and ventricular fibrillation can all result from hypokalemia.

Patients on digoxin are at increased risk for hypokalemia-associated arrhythmia. Fatal arrhythmia can occur with IV potassium repletion over 40 mEq/h.

Magnesium levels should be monitored in the setting of hypokalemia as magnesium is an important cofactor for the uptake and maintenance of potassium levels.

Supplemental magnesium also reduces the risk of cardiac arrhythmia.

How well did you know this?
1
Not at all
2
3
4
5
Perfectly
42
Q

A 70-kg man is nil per os and receiving maintenance intravenous fluids in the form of 5% dextrose in 0.45% saline after gastrointestinal surgery. Which of the following is true regarding his fluid and electrolyte requirements?

A. His daily water need is 4000 mL.

B. His sodium requirement is 1 g/day.

C. His potassium requirement is 50 mEq/day.

D. Average urine volume is 3000 mL.

E. If he were febrile, his average increase in insensible loss would be 250 mL/day for each degree of fever.

A

ANSWER: E

The average individual has an intake of 2000 to 2500 mL of water per day—1500 mL is ingested orally, and the remainder is acquired in solid food. Daily losses include 250 mL in stool, 800 to 1500 mL in urine, and approximately 600 mL as insensible loss. To excrete the products of normal daily catabolism, an individual must produce at least 500 to 800 mL of urine. In healthy individuals, 75% of insensible loss occurs through the skin and 25% through the lungs. Insensible loss from the skin occurs as loss of water vapor through the skin and not by evaporation of water secreted by the sweat glands. In febrile patients, insensible loss through the skin may increase to 250 mL/day for each degree of fever. Losses from sweating can be as high as 4 L/h. In a patient with a tracheostomy who is being ventilated with unhumidified air, insensible loss from the lungs may increase to 1500 mL/day. Average sodium needs are 2 to 4 g/day. Maintenance of potassium requires 100 mEq/ day.

How well did you know this?
1
Not at all
2
3
4
5
Perfectly
43
Q

Which of the following statements regarding hypervolemia in postoperative patients is true?

A. Hypervolemia can be produced by the administration of isotonic salt solutions in amounts that exceed the loss of volume.

B. Acute overexpansion of the ECF space is typically not well tolerated in healthy individuals.

C. Excess administration of normal saline can result in metabolic derangement, most commonly hyperchloremic metabolic alkalosis.

D. The most reliable sign of volume excess is peripheral edema.

E. Daily weight measurement in the postoperative period does not help determine fluid status.

A

ANSWER: A

COMMENTS: The earliest sign of volume excess during the postoperative period is weight gain. Normally, during this period the patient is in a catabolic state and is expected to lose weight (1/4 to 1/2 lb/day).

Circulatory and pulmonary signs of overload appear late and usually represent massive overload.

Peripheral edema does not necessarily indicate excess volume. In a patient with edema but without additional evidence of volume overload, other causes of peripheral edema should be considered.

The most common cause of excess volume in a surgical patient is the administration of isotonic salt solutions in amounts that exceed the loss of volume. In a healthy individual, such overload is usually well tolerated. However, if the excess fluid is administered for several days, the ability of the kidneys to secrete sodium may be exceeded, thus resulting in hypernatremia.

In the case of excess normal saline administration, hyperchloremic metabolic acidosis may occur.

How well did you know this?
1
Not at all
2
3
4
5
Perfectly
44
Q

With regard to potassium, which of the following statements is true?

A. Normal dietary intake of potassium is 150 to 200 mEq/day.

B. In patients with normal renal function, most ingested potassium remains in the extracellular space.

C. More than 90% of the potassium in the body is located in the extracellular compartment.

D. Critical hyperkalemia (>6 mEq/L) is rarely encountered if renal function is normal.

E. Administration of sodium bicarbonate shifts potassium from the ICF to the ECF.

A

ANSWER: D

COMMENTS: The average daily dietary intake of potassium is 50 to 100 mEq. In patients with normal renal function and serum potassium levels, most ingested potassium is excreted in urine.

More than 90% of the body’s potassium stores are within the intracellular compartment at a concentration of 150 mEq/L.

Although the total extracellular potassium concentration is just 50 to 70 mEq (4.5 mEq/L), this concentration is critical for cardiac and neuromuscular function.

Significant quantities of intracellular potassium are released in response to severe injury, surgical stress, acidosis, and a catabolic state.

However, dangerous hyperkalemia (>6 mEq/L) is rarely encountered if renal function is normal.

The administration of bicarbonate shifts potassium from the ECF across the cell membrane into the ICF.

How well did you know this?
1
Not at all
2
3
4
5
Perfectly
45
Q

Which of the following electrocardiographic (ECG) findings is associated with hyperkalemia?

A. Inverted T waves

B. Shortened PR interval

C. Peaked P waves

D. Narrowing of the QRS complex

E. T waves higher than R waves in more than one lead

A

ANSWER: E

COMMENTS: Hyperkalemia occurs when the serum potassium level exceeds 5 mmol/L. As potassium increases, changes in the resting membrane potential of cells impair depolarization and repolarization and lead to cardiac arrhythmias.

The signs of hyperkalemia are generally limited to cardiovascular and gastrointestinal symptoms.

Gastrointestinal symptoms include nausea, vomiting, intermittent intestinal colic, and diarrhea.

ECG changes can be the first manifestation of hyperkalemia and include peaked T waves and a prolonged PR interval, which are characteristic early findings.

These ECG changes may be seen with potassium concentrations greater than 6 mEq/L. Symmetrically peaked T waves indicate dangerous hyperkalemia, particularly if the T waves are higher than the R wave in more than one lead.

At higher potassium concentrations (7 mmol/L), loss of P waves, slurring, or widening of the QRS complexes occurs.

As [K+] exceeds 8 mmol/L, sudden lethal arrhythmias, such as asystole, ventricular fibrillation, or a wide pulseless idioventricular rhythm, ensue.

How well did you know this?
1
Not at all
2
3
4
5
Perfectly
46
Q

With regard to postoperative hyponatremia, which of the
following statements is true?

A. It does not occur when water is used to replace sodium- containing fluids because intracellular reserves often replace these losses.

B. In patients with head injury, hyponatremia despite adequate salt administration is usually caused by occult renal dysfunction.

C. In oliguric patients, cellular catabolism with resultant metabolic acidosis increases cellular release of water and can contribute to hyponatremia.

D. Hyperglycemia is not a cause of hyponatremia.

E. Patients with salt-wasting nephropathy usually have
abnormal blood urea nitrogen and creatinine values.

A

ANSWER: C

COMMENTS: Abnormalities in sodium concentration do not usually occur during the postoperative period if the functional ECF volume has been adequately replaced during the operation.

The sodium concentration generally remains normal because the kidneys retain the ability to excrete moderate excesses of water and solute administered during the early postoperative period.

Hyponatremia does occur when water is given to replace lost sodium-containing fluids or when the amount of water given consistently exceeds the amount of water lost.

In patients with head injury, hyponatremia may develop despite adequate salt administration because of excessive secretion of antidiuretic hormone (ADH) with resultant increased water retention.

Patients with preexisting renal disease and loss of concentrating ability may excrete urine with a high salt concentration. This salt- wasting phenomenon is commonly encountered in elderly patients and is often not anticipated because the blood urea nitrogen and creatinine levels are within normal limits.

When there is doubt, determination of the urine sodium concentration can help clarify the diagnosis.

Oliguria reduces the daily water requirement and can lead to hyponatremia if not anticipated.

Cellular catabolism in patients without adequate caloric intake can lead to gain of significant quantities of water released from the tissues.

Hyperglycemia may produce a depressed serum sodium level by exerting an osmotic force in the extracellular compartment and thus diluting serum sodium levels.

How well did you know this?
1
Not at all
2
3
4
5
Perfectly
47
Q
  1. Which of the following statements regarding changes in volume status of the ECF compartment is true?

A. Hyponatremia is diagnostic of excess ECF volume.

B. Hypernatremia is diagnostic of depletion of ECF volume.

C. Excess extracellular volume
is usually iatrogenic or due to renal or cardiac failure.

D. Central nervous system symptoms appear after tissue signs with acute volume loss.

E. The concentration of serum sodium is directly related to extracellular volume.

A

ANSWER: C

COMMENTS: The serum concentration of sodium is not neces- sarily related to the volume status of the ECF compartment. Volume deficit or excess can exist with high, low, or normal serum sodium concentrations.

Volume deficit is the most frequent volume disorder encountered during surgery. Its most common cause is loss of isotonic fluid (i.e., fluid having the same composition as ECF), for example, through hemorrhage, vomiting, diarrhea, fistulas, or third spacing.

With acute volume loss, central nervous system symptoms (e.g., sleepiness and apathy progressing to coma) and cardiovascu- lar signs (e.g., orthostasis, hypotension, tachycardia, and coolness in the extremities) appear first, along with decreasing urine output.

Tissue signs (e.g., decreased turgor, softness of the tongue with longitudinal wrinkling, and atonicity of muscles) usually do not appear during the first 24h.

In response to hypovolemia, body temperature may be slightly decreased. It is therefore important to also monitor the body temperature of hypovolemic patients.

Signs and symptoms of sepsis may be depressed in volume-depleted patients. The abdominal pain, fever, and leukocytosis associated with peritonitis may be absent until ECF volume is restored.

Volume overload is generally either iatrogenic or the result of renal insufficiency or heart failure. Both plasma and the interstitial fluid spaces are involved.

The signs are those of circulatory overload and include distended veins, bounding pulses, functional murmurs, edema, and basilar rales.

These signs may be present in young, healthy patients, but these patients can compensate for moderate to severe volume excess without overt failure or pulmonary edema development.

In elderly patients, however, congestive heart failure (CHF) with pulmonary edema may develop quite rapidly.

How well did you know this?
1
Not at all
2
3
4
5
Perfectly
48
Q

With regard to hypokalemia, which of the following
statements is true?

A. It is less common than hyperkalemia in surgical patients.

B. Respiratory acidosis is associated with increased renal potassium loss.

C. Hypokalemia can cause increased deep tendon reflexes.

D. Flattened T waves and a prolonged QT interval are associated with hypokalemia.

E. Intravenous potassium administration should not exceed 10 to 20 mEq/h.

A

ANSWER: D

COMMENTS: Hypokalemia is more common than hyperkalemia in surgical patients.

Hypokalemia can result from increased renal excretion, prolonged administration of potassium-free fluids, hyper- alimentation with inadequate potassium replacement, or gastrointestinal losses.

Respiratory and metabolic alkalosis results in increased renal potassium loss because potassium is preferentially excreted in an attempt to preserve hydrogen ions.

Loss of gastrointestinal secretions can also be a significant cause of potassium depletion. This problem is compounded if potassium-free fluids are used for volume replacement.

Signs of hypokalemia, including paralytic ileus, diminished or absent tendon reflexes, weakness, and even flaccid paralysis, are related to decreased muscle contractility.

ECG changes include flattened or inverted T waves, U waves, and prolongation of the QT interval. The best treatment of hypokalemia is prevention.

Gastrointestinal losses should be treated by the administration of fluids containing enough potassium to replace the daily obligatory loss (20 mEq/day), as well as the additional losses in gastrointestinal drainage.

As a rule, no more than 40 to 60 mEq of potassium should be added to each liter of intravenous fluid, and the rate of potassium administration should never exceed 40 to 60 mEq/h.

How well did you know this?
1
Not at all
2
3
4
5
Perfectly
49
Q

With regard to abnormalities in serum sodium concentration, which of the following statements is true?

A. Changes in serum sodium concentration usually produce changes in the status of ECF volume.

B. The chloride ion is the main determinant of the osmolarity of the ECF space.

C. Extracellular hyponatremia leads to depletion of intracellular water.

D. Dry, sticky mucous membranes are characteristic of hyponatremia.

E. Preservation of normal ECF has higher precedence than maintenance of normal osmolality.

A

ANSWER: A

COMMENTS: Although extracellular volume may change without a change in serum sodium concentration (as occurs after isotonic volume losses), changes in serum sodium concentration usually produce changes in ECF volume because the serum sodium concentration is the main determinant of the osmolarity of the ECF space.

Alterations in its concentration produce concomitant shifts in water volume. Signs and symptoms of hypernatremia and hyponatremia are not generally present unless the changes are severe or the alteration in sodium concentration occurs rapidly.

Hyponatremia is caused by excessive intake of hypotonic fluids or salt loss that exceeds water loss. With hyponatremia, decreased extracellular osmolarity causes a shift of water into the intracellular compartment. When such a shift occurs, central nervous system symptoms caused by increased intracranial pressure develop, and tissue signs of excess water are noted.

Central nervous system symptoms include muscle twitching, hyperactive tendon reflexes, and, when the hyponatremia is severe, convulsions and hypertension. Tissue signs include salivation, lacrimation, watery diarrhea, and “fingerprinting” of the skin.

When hyponatremia develops rapidly, signs and symptoms may appear at sodium concentrations of less than 130 mEq/L. Acute dilution of osmolality can occur if patients with an ECF deficit are given sodium-free water.

The hyponatremia is exacerbated in hypovolemic patients because of secretion of ADH as a result of the hypothalamic–pituitary response to both elevated ECF osmolality and a reduction in ECF volume.

The normal response of the hypothalamic–pituitary axis to hyponatremia is suppression of ADH release, and as the dilute urine is excreted, there is a corrective increase in serum [Na+].

A moderate or severely hyponatremic patient should have undetectable blood levels of ADH.

Preservation of normal ECF has higher precedence than maintenance of normal osmolality.

In symptomatic patients, administration of hypertonic (3%) solutions of sodium may be indicated to correct the problem in those with severe hyponatremia who are at risk for seizures.

In less severe cases, restriction of free water and judicious infusion of normal saline solution are usually sufficient. In patients with acute hyponatremia and [Na+] less than 120 mEq/L, the rate of infusion of sodium-containing solutions should not increase serum [Na+] more rapidly than 0.25 mEq/L/h.

Chronic hyponatremia develops slowly, and patients may have sodium levels as low as 120 mEq/L before becoming symptomatic.

Severe hyponatremia may be associated with the onset of irreversible oliguric renal failure. Patients with a closed head injury are sensitive to even mild hyponatremia because of increased intracellular water, which exacerbates the increased intracranial pressure associated with the head injury.

The syndrome of inappropriate release of antidiuretic hormone (SIADH) and chronic renal failure are frequent causes of hyponatremia. The diagnosis of SIADH can be made only in euvolemic patients who have a serum osmolality of less than 270 mmol/kg H2O along with inappropriately concentrated urine.

Hypernatremia is the result of excessive free water loss or salt intake. Central nervous system signs and symptoms associated with hypernatremia include restlessness, weakness, delirium, and maniacal behavior.

The tissue signs are characteristic and include dryness and stickiness of mucous membranes, decreased salivation and tear production, and redness and swelling of the tongue. Body temperature is usually elevated, occasionally to a lethal level.

An acute onset of hypernatremia increases ECF osmolality and contracts the size of the ICF compartment.

Patients have moderate hypernatremia if their serum [Na+] is 146 to 159 mEq/L.

Water loss is the most common explanation for acute hypernatremia. Neurologic damage as a result of contraction of brain cell volume is the primary risk associated with hypernatremia.

Patients with diabetes insipidus or nephrogenic diabetes insipidus have a failure to synthesize and release ADH or a failure of the renal tubular cells to respond to ADH, respectively, thus leading to hypernatremia.

Treatment of patients with hypernatremia secondary to dehydration involves the administration of water. Hypernatremic patients are frequently hypovolemic, and these patients are treated by the intravenous infusion of isotonic saline solution until the volume deficit has been restored.

A rapid decline in ECF osmolality in a severely hypernatremic patient can lead to cerebral injury as a result of cellular swelling. [Na+] should be lowered at a rate not to exceed 8 mEq/day (Table 4.1).

Patients with central diabetes insipidus are treated with desmopressin [1-desamino-8-D-arginine vasopressin (DDAVP)]. Desmopressin is a synthetic analogue of ADH.

How well did you know this?
1
Not at all
2
3
4
5
Perfectly
50
Q

A 45-year-old alcoholic man is found to have hypomagnesemia. Which of the following statements about magnesium is true?

A. The distribution of nonosseous magnesium is similar to that of sodium.

B. Calcium deficiency cannot be adequately corrected until the hypomagnesemia is addressed.

C. Magnesium depletion is characterized by depression of the neuromuscular and central nervous systems.

D. Magnesium supplementation should be stopped as soon as the serum level has normalized.

E. The treatment of choice for magnesium deficiency is oral magnesium phosphate.

A

ANSWER: B

COMMENTS: The body contains 2000 mEq of magnesium, half of which is contained in bone. Most of the remaining magnesium is intracellular (a distribution similar to that of potassium).

Plasma levels range between 1.5 and 2.5 mEq/L. Normal dietary intake is 240 mg/day, most of which is excreted in feces.

The kidneys excrete some magnesium but can help conserve magnesium when a deficiency is present.

Hypomagnesemia (like calcium deficiency) is characterized by neuromuscular and central nervous system hyperactivity. Hypomagnesemia can occur with starvation, malabsorption, protracted loss of gastrointestinal fluid, and prolonged parenteral therapy without proper magnesium supplementation. An accompanying calcium deficiency, cannot be successfully treated until the hypomagnesemia is corrected.

Magnesium deficiency is treated with parenteral administration of magnesium sulfate or magnesium chloride. The extracellular magnesium concentration can be restored rapidly, but therapy must be continued for 1 to 2 weeks to replenish the intracellular component.

To avoid magnesium deficiency, patients managed with hyperalimentation should receive 12 to 24 mEq of magnesium daily. Oral supplementation and intramuscular injection are alternative routes for replacement but are not preferred.

Magnesium toxicity is rare except in the setting of renal insufficiency. Immediate treatment is infusion of calcium chloride or calcium gluconate; if the symptoms persist, dialysis may be required.

How well did you know this?
1
Not at all
2
3
4
5
Perfectly
51
Q

Which of the following clinical situations can be associated with hypovolemic hyponatremia?

A. CHF

B. SIADH

C. Cirrhosis

D. Hyperglycemia

E. Gastrointestinal losses

A

ANSWER: E

COMMENTS: Hyponatremia in a surgical patient can be clas- sified into hypervolemic, euvolemic, and hypovolemic categories, which can then be further subclassified according to tonicity (hypertonic, >290 mOsm; isotonic, 280 to 290 mOsm; and hypotonic, <280 mOsm).

For simplicity and rapid clinical evaluation, volume status can be used to direct treatment. Hypervolemic hyponatremia may be caused by increased intake of water, post-operative secretion of ADH, and high ECF volume states such as cirrhosis and CHF.

Hyponatremia can develop in patients with edema and ascites secondary to CHF, nephrotic syndrome, or cir- rhosis despite having an expanded overall volume of extracellular water. These patients have an excess of sodium but an even greater proportional increase in water volume. Their pathophysiologic condition entails an overall contracted intravascular volume, which stimulates the release of vasopressin from the hypothalamus centrally.

Peripherally, renal hypoperfusion contributes to water retention. Fluid restriction is crucial to the treatment of this type of hyponatremia.

In patients with severe hyponatremia, small volumes of hypertonic saline solution may be administered. Diure- sis may be used but is generally unsuccessful. Hemodialysis may be performed in extreme circumstances of fluid excess.

Euvolemic hyponatremia may be caused by hyperglycemia, hyperlipidemia, or hyperproteinemia (termed pseudohyponatremia because of relative hyperosmolar protein, lipid, or glucose-rich plasma drawing fluid from the interstitial space and diluting plasma sodium), SIADH, water intoxication, and diuretics. SIADH is characterized by functional reabsorption of free water and subsequent dilution of plasma sodium.

Hypovolemic hyponatremia may be caused by decreased overall sodium intake, gastrointestinal losses, renal losses associated with the use of diuretics (especially thiazide diuretics), and primary renal disease.

Conversely, hypernatremia can also be subdivided into volume states. Hypervolemic hypernatremia may be caused by iatrogenic sodium administration or mineralocorticoid excess (e.g., aldosteronism, Cushing disease, congenital adrenal hyperplasia).

Euvolemic hypernatremia may be associated with renal (renal disease, diuretics, or diabetes insipidus) or nonrenal free water loss through the skin or gastrointestinal tract.

Hypovolemic hypernatremia can likewise be subdivided into nonrenal and renal water loss.

How well did you know this?
1
Not at all
2
3
4
5
Perfectly
52
Q

With regard to intraoperative management of fluids, which of the following statements is true?

A. In a healthy person, up to 500 mL of blood loss may be well tolerated without the need for blood replacement.

B. During an operation, functional ECF volume is directly related to the volume lost to suction.

C. Functional ECF losses should be replaced with plasma.

D. Administration of albumin plays an important role in the replacement of functional ECF volume loss.

E. Operative blood loss is usually overestimated by the surgeon.

A

ANSWER: A

COMMENTS: It is now believed that the routine use of albumin to replace blood and ECF losses intraoperatively is not indicated and may be potentially harmful.

Maintenance of cardiac and pulmonary function by replacing blood with blood products and ECF with “mimic” solutions can be achieved without the addition of albumin. In general, it is believed that blood should be replaced as it is lost. However, it is usually unnecessary to replace blood loss of less than 500 mL.

Operative blood loss is usually underestimated by the surgeon by 15%–40% in comparison to the isotopically measured loss, a factor that may contribute to the detection of anemia during the immediate postoperative period.

How well did you know this?
1
Not at all
2
3
4
5
Perfectly
53
Q

Which of the following statements regarding total body water is true?

A. In males, approximately 40% of total body weight is water.

B. The percentage of total body weight that is water is higher in females than in males.

C. Obese individuals have a greater proportion of water (relative to body weight) than lean individuals.

D. The percentage of total body water decreases with age.

E. The majority of body water is contained within the
interstitial fluid compartments.

A

ANSWER: D

COMMENTS: Approximately 50%–75% of body weight is water. In males, 60% (±15%) of body weight is water, and in females, 50% (±15%) of body weight is water. Age and lean body mass also contribute to differences in the percentage of total body weight that is water.

Since fat contains little water, lean individuals have a greater proportion of body water than obese individuals of the same weight. Because females have more subcutaneous fat in relation to lean mass than males, they have less body water. Total body water decreases with age as a result of decreasing lean muscle mass. Infants have an unusually high ratio of total body water to body weight: up to 75%–80%. By 1 year of age, however, the percentage of body water approaches that of adults.

Body water is divided into three functional compartments: the ICF compartment (40% of body weight) and the ECF compart- ment (20% of body weight), which is further subdivided into the interstitial (15% of body weight) and intravascular (5% of body weight) fluid compartments.

How well did you know this?
1
Not at all
2
3
4
5
Perfectly
54
Q

A 62-year-old female takes 40 mg of furosemide twice daily for hypertension and CHF. Which of the following is true?

A. Loop diuretics act on the distal convoluted tubule in the nephron.

B. Magnesium is affected by loop diuretics.

C. Fatigue and muscle weakness are not side effects of her medication.

D. Loop diuretics decrease venous capacitance.

E. Loop diuretics are agonists to the sodium-potassium-
chloride cotransporter.

A

ANSWER: B

COMMENTS: Loop diuretics, such as furosemide, are potent inhibitors of the sodium-potassium-chloride cotransporter. They act by competing for the chloride-binding site at the thick ascending limb of the loop of Henle. The effect is inhibition of sodium reabsorption, resulting in diuresis. Magnesium, potassium, and calcium will likewise be excreted, with the net increase in urine output.

Hypomagnesemia can result in fatigue, muscle weakness, numbness, or even convulsions. Therefore it is important to monitor serum levels to prevent depletion while a patient is being treated with a loop diuretic.

Loop diuretics are commonly used for pulmonary edema because of their potency. In addition to inhibition of sodium absorption, they increase blood flow to the kidneys by stimulating vasodilatory prostaglandins and increase venous capacitance, which can quickly relieve pulmonary edema, even before diuresis and natriuresis have occurred.

These three mechanisms help decrease ECF volume. Loop diuretics, such as furosemide or bumetanide, are extensively protein bound and must reach their intratubular site of action through active proximal tubular secretion.

How well did you know this?
1
Not at all
2
3
4
5
Perfectly
55
Q

With regard to distributional shifts during an operation,
which of the following statements is true?

A. The surface area of the peritoneum is not large enough to account for significant third-space loss.

B. Approximately 1 to 1.5 L/h of fluid is needed during an operation.

C. Blood is replaced as it is lost, without modification of the basal operative fluid replacement rate.

D. Sequestered ECF is predominantly hypotonic.

E. A major stimulus to ECF expansion is peripheral
vasoconstriction.

A

ANSWER: C

COMMENTS: The functional ECF volume decreases during major abdominal operations largely because of sequestration of fluid in the operative site as a consequence of (1) extensive dissection, (2) fluid collection within the lumen and wall of the small bowel, and (3) accumulation of fluid in the peritoneal cavity.

The surface area of the peritoneum is 1.8 m2. When irritated, it can account for a functional loss of several liters of fluid that is not readily apparent. It is generally agreed that this lost volume should be replaced during the course of an operation with an isotonic saline solution as a “mimic” of sequestered ECF.

Although there is no set formula for intraoperative fluid therapy, useful guidelines for replacement include the following.

(1) Blood is replaced as it is lost, regardless of additional fluid therapy, provided that the patient meets the criteria for transfusion: hemoglobin concentration less than 7 g/dL.
(2) Lost ECF should be replaced during the operative procedure if there is a delay in replacement until after the operation; fluid management is then complicated by adrenal and hypophyseal compensatory mechanisms that respond to operative trauma during the immediate postoperative period.

How well did you know this?
1
Not at all
2
3
4
5
Perfectly
56
Q

With regard to perioperative fluid management, which of the following statements is correct?

A. Insensible loss is approximately 600 mL/day.

B. Intraoperative insensible losses from an open abdomen are less than 250 mL/h.

C. About 200 to 300 mL of fluid is needed to excrete the catabolic end products of metabolism.

D. Lost urine should be replaced milliliter for milliliter.

E. Hypermetabolism and hyperventilation are not important
factors in postoperative fluid loss or management.

A

ANSWER: A

COMMENTS: Postoperative fluid management requires assessment of the patient’s volume status and evaluation for possible disorders in concentration or composition. All measured and insensible losses should be treated by replacement with appropriate fluids.

In patients with normal renal function, the amount of potassium given is 40 mEq/day for replacement of renal excretion. An additional 20 mEq should be given for each liter of gastrointestinal loss.

Insensible water loss is usually constant in the range of 600 mL/day. It can be increased to 1500 mL/day by hypermetabolism, hyperventilation, or fever. Insensible loss is replaced with 5% dextrose in water. Insensible loss may be offset by an insensible gain of water from excessive catabolism in postoperative patients who require prolonged intravenous fluid therapy.

Approximately 800 to 1000 mL/day of fluid is needed to excrete the catabolic end products of metabolism. Because the kidneys are able to conserve sodium in a healthy individual, this amount can be replaced with 5% dextrose in water. A small amount of salt is usually added, however, to relieve the kidneys of the stress of sodium resorption.

If there is a question regarding urinary sodium loss, measurement of urinary sodium levels helps determine the type of fluid that can best be used.

Urine volume should not be replaced milliliter for milliliter because high output may represent diuresis of the fluids given during surgery or the diuresis that takes place to eliminate excessive fluid administration.

Sensible or measurable losses such as those from the gastrointestinal tract are usually isotonic and should therefore be treated by replacement in equal volumes with isotonic salt solutions. The type of salt solution selected depends on determination of the patient’s serum sodium, potassium, and chloride levels.

In general, replacement fluids are administered at a steady rate over a period of 18 to 24 h as losses are incurred.

How well did you know this?
1
Not at all
2
3
4
5
Perfectly
57
Q

Which of the following has no effect on the development of hypernatremia?

A. Excessive sweating

B. Hyperlipidemia

C. Lactulose

D. Glycosuria

E. Inadequate maintenance fluids

A

ANSWER: B

COMMENTS: Hypernatremia is less common than hyponatremia in postoperative patients and is a reflection of elevated serum osmolality and hypertonicity. It is indicative of a deficiency of free water relative to the sodium concentration.

Decreased intake of water, increased loss of water, and increased intake of sodium are the main mechanisms responsible for the development of hypernatremia. Loss of the thirst mechanism and an inability to access free water are mechanisms by which hypernatremia secondary to decreased intake of water can develop.

Excessive sweating and large evaporative losses are mechanisms of loss of free water. Agents such as lactulose and sorbitol and carbohydrate malabsorption can cause osmotic diarrhea and result in relative losses of hypotonic fluid. Similarly, hyperglycemia causing glycosuria or diuresis in a catabolic patient excreting excess urea can also cause an osmotic diuresis.

Both hyperlipidemia and hyperproteinemia are responsible for an entity known as pseudohyponatremia, which occurs when excess lipids or proteins displace water and create a falsely measured hyponatremia.

How well did you know this?
1
Not at all
2
3
4
5
Perfectly
58
Q

With regard to diabetes insipidus, which of the following
statements is true?

A. Diabetes insipidus causes hypervolemic hyponatremia.

B. Central diabetes insipidus cannot be corrected by the administration of desmopressin.

C. Treatment of diabetes insipidus requires correction of hypernatremia at a rate faster than 12 mEq/day.

D. Alcohol intoxication can mimic diabetes insipidus.

E. Lithium administration could induce central diabetes
insipidus.

A

ANSWER: D

COMMENTS: Diabetes insipidus is one of the causes of hypovolemic hypernatremia and is marked by a continual production of dilute urine of less than 200 mOsm/kg H2O in the context of serum osmolarity of ECF greater than 300 Osm/L.

Patients can have either central (lack of production of ADH by the hypothalamus) or nephrogenic (lack of response of the distal tubule of the nephron to ADH) diabetes insipidus.

Alcohol causes suppression of vasopressin release and can mimic central diabetes insipidus.

Treatment of hypernatremia consists of slow correction of sodium by the administration of free water. Whenever hypernatremia develops, a relative free water deficit exists and must be replaced.

The water deficit can be approximated by using the following formula:

water deficit = total body water × (1 − 140 ÷ serum sodium)

Usually, the rate of correction of hypernatremia should not exceed 12 mEq/L/day. The aim should be to correct approximately half the deficit over the first 24 h. Too rapid correction of hypernatremia may lead to cerebral edema and seizures.

Desmopressin is a synthetic analogue of ADH that can be used to mimic arginine vasopressin (AVP) and to differentiate between central and nephrogenic diabetes insipidus. It is the agent of choice for treating patients with central diabetes insipidus because the drug increases water movement out of the collecting duct but does not have the vasoconstrictive effects of ADH.

Central diabetes insipidus will respond to desmopressin, whereas nephrogenic diabetes insipidus will not. Unlike vasopressin, desmopressin is only renally active and does not have the vasoactive side effects.

Lithium and amphotericin B can induce nephrogenic, not central, diabetes insipidus.

How well did you know this?
1
Not at all
2
3
4
5
Perfectly
59
Q

A postoperative patient has a serum sodium concentration of 125 mEq/L and a blood glucose level of 500 mg/dL (normal level, 100 mg/dL). What would the patient’s serum sodium concentration be (assuming normal renal function and appropriate intraoperative fluid therapy) if blood glucose levels were normal?

A. 120 mEq/L

B. 122 mEq/L

C. 137 mEq/L

D. 142 mEq/L

E. 147 mEq/L

A

ANSWER: C

COMMENTS: Serum osmolality is described as the amount of solutes per unit of water. It can be measured with an osmometer, or it can be calculated. It is reported as milliosmoles per liter. Cal- culation of serum osmolality is performed with the following equation:

P(osm)
= 2 × Na (mEq/L) + Glucose(mg) / 18 + BUN(mg) / 2.9

The serum concentrations of sodium, urea, and glucose are required, whereas that of chloride is not required for the calculation. Simply doubling the serum sodium concentration provides an adequate estimate of serum osmolality.

As a general rule, each 100-mg/dL rise in the blood glucose level above normal is equivalent to a 1.6- to 3.0-mEq/L fall in the apparent serum sodium concentration.

For example, if the patient has a blood glucose level of 500 mg/dL, or 400 mg/dL above normal, this is equivalent to a 12-mEq/L change in the serum sodium level.

If this patient has a measured sodium concentration of 125 mEq/L, the sodium concentration is actually 137 mEq/L once the excess extracellular water has been eliminated.

How well did you know this?
1
Not at all
2
3
4
5
Perfectly
60
Q

Which one of the following is least useful in the immediate treatment of hyperkalemia?

A. Calcium salts

B. Sodium bicarbonate

C. Potassium-binding resins

D. Glucose and insulin

E. Hemodialysis

A

ANSWER: C

COMMENTS: The most dreaded complication of hyperkalemia is the development of a lethal arrhythmia. Immediate management includes ECG monitoring and cessation of all potassium supplementation and potassium-sparing drugs.

Calcium is administered intravenously to stabilize the membrane potential and decrease myocardial excitability. It acts in less than 5 min, and the effects last for 30 to 60 min.

Sodium bicarbonate drives potassium into cells, thereby transiently reducing serum potassium levels. Its actions last 15 to 30 min. Insulin and glucose also facilitate entry of potassium into cells, with an almost immediate onset of action.

In cases of severe hyperkalemia, hemodialysis is the definitive and most rapid method of decreasing extracellular potassium.

Potassium-binding resins, such as sodium polystyrene sulfonate (Kayexalate), begin lowering serum potassium within 1 to 2 h and last 4 to 6 h. Rectal administration of these binding resins is more effective than oral formulations. However, enemas with sodium polystyrene sulfonate combined with sorbitol have been associated with colon necrosis and perforation.

Kaliuresis through the administration of diuretics such as acetazolamide is also effective in reducing serum potassium levels.

How well did you know this?
1
Not at all
2
3
4
5
Perfectly
61
Q

Which one of the following is not associated with
hypocalcemia?

A. Shortening of the QT interval

B. Painful muscle spasms

C. Perioral or fingertip tingling

D. Seizures in children

E. Prolongation of the QT interval

A

ANSWER: A

COMMENTS: The symptoms of hypocalcemia are generally seen at serum levels of less than 8 mg/dL.

Symptoms include numbness and tingling in the circumoral area and in the tips of the fingers and toes.

Signs include hyperactive deep tendon reflexes, positive Chvostek sign, positive Trousseau sign, muscle and abdominal cramps, tetany with carpal pedal spasm, or convulsions.

The ECG may show prolongation of the QT interval. Calcium is found in three forms in the body: protein bound (≈50%, mostly to albumin); diffusible calcium combined with anions such as bicarbonate, phosphate, and acetate (5%); and ionized (≈45%).

Patients with severe alkalosis may have symptoms of hypocalcemia despite normal serum calcium levels because the ionized calcium is markedly decreased.

Conversely, hypocalcemia without signs or symptoms may be present in patients with hypoproteinemia and a normal ionized fraction.

Acute symptoms can be relieved by the intravenous administration of calcium gluconate or calcium chloride.

Patients requiring prolonged replacement can be treated with oral calcium, often given with vitamin D.

How well did you know this?
1
Not at all
2
3
4
5
Perfectly
62
Q

Which one of the following clinical signs or symptoms is associated with serum sodium concentrations below 125 mEq/L?

A. Restlessness

B. Hallucinations

C. Tachycardia

D. Hyperventilation

E. Hyperthermia

A

ANSWER: B

COMMENTS: In most patients with symptomatic hyponatremia, the serum sodium concentration decreases below 125 mEq/L.

When the concentration falls below 125 mEq/L, clinical signs and symptoms may occur, including headache, nausea, lethargy, hallucinations, seizures, bradycardia, hypoventilation, and occasionally coma. Hypothermia, not hyperthermia, occurs.

How well did you know this?
1
Not at all
2
3
4
5
Perfectly
63
Q

Which one of the following is not a stimulus for ECF
expansion?

A. Hemorrhage leading to a reduction in blood volume

B. Increased capillary permeability after major surgery

C. Peripheral arterial vasoconstriction

D. Negative interstitial fluid hydrostatic pressure

E. Colloid oncotic pressure

A

ANSWER: C

COMMENTS: Approximately 85% of the ECF that is within the vascular compartment resides in the venous circulation. Therefore the remaining 15% resides within the arterial system.

The vascular compartment, otherwise known as plasma fluid, constitutes approximately one-third of the ECF.

Interstitial fluid (i.e., fluid between the cells) makes up approximately two-thirds of the ECF.

The ECF constitutes one-third of total body water, whereas the ICF represents two-thirds.

Expansion of ECF is primarily driven by three mechanisms, all of which have the final common stimulus of reduction of intravascular volume. The first mechanism, hemorrhage, is directly responsible for the reduction in blood volume. Through various pathways, this drop in volume signals the retention and sequestration of fluid in the intravascular space.

Increased capillary permeability, the second mechanism, occurs following major surgery and is due to the loss of endothelial integrity. This loss of integrity is mediated by several humoral factors that act on the endothelium. The end result of the loss of endothelial integrity is extravasation of protein-rich fluid into the interstitium, with a consequent increase in the interstitial fluid space.

This constitutes the third mechanism of ECF expansion. Serum albumin is a major determinant of colloid oncotic pressure, and hypoalbuminemia could lead to transudation of fluid from the vascular to the interstitial compartment.

This concept is expressed mathematically by the Starling equation: Qf = Kf × (Pv − Pt) − δ × (COP − TOP), where Qf is fluid flux, Kf is the capillary filtration coefficient, Pv is vascular hydrostatic pressure, Pt is interstitial hydrostatic pressure, δ is a reflection coefficient (which defines the effectiveness of the membrane in preventing flow of solutes), COP is colloid osmotic pressure, and TOP is tissue osmotic pressure.

How well did you know this?
1
Not at all
2
3
4
5
Perfectly
64
Q

A 70-year-old man with sepsis has a pH of 7.18. Which of the following statements is true regarding his metabolic acidosis?

A. Tissue hypoxia leads to increased oxidative metabolism.

B. Acute compensation for metabolic acidosis is primarily renal.

C. Metabolic acidosis results from the loss of bicarbonate or the gain of fixed acids.

D. The most common cause of excess acid is prolonged nasogastric suction.

E. Restoration of blood pressure with vasopressors corrects the metabolic acidosis associated with circulatory failure.

A

ANSWER: C

COMMENTS: Metabolic acidosis results from the retention or gain of fixed acids (e.g., through diabetic acidosis or lactic acidosis) or the loss of bicarbonate (e.g., through diarrhea, small bowel fistula, or renal tubular dysfunction).

Initial compensation is respiratory (by hyperventilation). Renal compensation is slower and occurs through the same means as the renal compensation for respiratory acidosis: excretion of acid salts and retention of bicar- bonate. This compensation depends on normal renal function. When kidney damage interferes with the ability to excrete acid and resorb bicarbonate, metabolic acidosis may rapidly progress to profound levels.

The most common cause of metabolic acidosis in surgical patients is circulatory failure, with tissue hypoxia and anaerobic metabolism, leading to the accumulation of lactic acid. Resuscitation with vasopressors or infusion of bicarbonate does not correct the underlying problem. Replacement of volume with a balanced electrolyte solution, blood, or both results in restoration of the circulation, hepatic clearance of lactate, consumption of the formed bicarbonate, and clearance of carbonic acid by the lung.

Excessive use of bicarbonate can cause metabolic alkalosis, which, in combination with other sequelae such as hypothermia and low levels of 2,3-diphosphoglycerate (from banked blood), shifts the oxygen–hemoglobin distribution curve to the left and thereby compromises oxygen delivery.

How well did you know this?
1
Not at all
2
3
4
5
Perfectly
65
Q

A 70-kg man with pyloric obstruction secondary to ulcer disease is admitted to the hospital for resuscitation after 1 week of prolonged vomiting. What metabolic disturbance is expected?

A. Hypokalemic, hyperchloremic metabolic acidosis

B. Hyperkalemic, hypochloremic metabolic alkalosis

C. Hyperkalemic, hyperchloremic metabolic acidosis

D. Hypokalemic, hypochloremic metabolic alkalosis

E. None of the above

A

ANSWER: D

COMMENTS: A common problem seen in patients with persistent emesis is hypokalemic, hypochloremic metabolic alkalosis.

To compensate for the alkalosis associated with the loss of chloride- and hydrogen ion–rich fluid from the stomach, bicarbonate excretion in urine is increased. The bicarbonate is usually excreted as a sodium salt. However, in an attempt to conserve intravascular volume, aldosterone-mediated sodium absorption occurs and leads to potassium and hydrogen excretion. This compounds the alkalosis and results in a paradoxical aciduria.

Management includes resuscitation with isotonic saline solutions and aggressive replacement of lost potassium.

How well did you know this?
1
Not at all
2
3
4
5
Perfectly
66
Q

A 49-year-old woman who has medically refractory
Crohn’s disease and high ileostomy output (2200 mL/
day) is referred to your clinic. She has undergone several abdominal surgeries for complications of her Crohn’s and she currently has a diverting ileostomy and
approximately 120 cm of her small bowel. Of note, she
has been unable to tolerate any inflammatory bowel dis
ease medications aside from corticosteroids.

The patient was admitted to an outside hospital twice
in the past 3 months for rehydration and repletion of
sodium, potassium, and magnesium. She now comes to
your institution complaining of increased ostomy output, lightheadedness, fatigue, and nausea. She reports a
recent weight loss of approximately 20 pounds (approximately 15% of total body weight). A Hickman catheter is placed in her right subclavian vein and she receives 3 days of parenteral nutrition without complications.

Insurance coverage for teduglutide is pending and she
is discharged home on parenteral nutrition. You would
like to perform an ileostomy takedown but would like
to improve her nutritional status first.

Compared to enteral nutrition, parenteral nutrition (PN):

A. Is less expensive

B. Does not suffer from product shortages

C. Preserves immunologic function of gut

D. Is not associated with metabolic bone dysfunction

E. Is less likely to cause diarrhea

A

E.

Compared to enteral nutrition, PN is less likely to
cause diarrhea.

Enteral nutrition is delivered directly to the GI tract and its hyperosmolarity may result in diarrhea, especially in patients with an underlying condition that causes malabsorption. Diarrhea has been shown to occur in as many as 95% of patients who receive enteral feeds.

PN is considerably more expensive than enteral nutrition. Shortages of many forms of product, especially vitamin and trace mineral components,has been causing delays in initiation of PN as well as inconsistent “mixing and matching” of different brands of product, which may result in certain micronutrient deficiencies if the provider
does not have expertise with PN.

Because it completely bypasses the GI tract, PN does not preserve the immunologic function of gut.

PN has been associated with metabolic bone dysfunction and abnormal bone metabolism and some patients have been shown to develop osteoporosis and osteomalacia.

How well did you know this?
1
Not at all
2
3
4
5
Perfectly
67
Q

Basic parenteral nutrition formulations include:

A. Sucrose

B. Amino acids

C. 30% IV fat emulsion

D. Omega-3 fatty acids

E. Insulin

A

B. Standard components of PN formulas include
amino acids, dextrose, a 10% or 20% IV fat emulsion
to provide essential fatty acids, electrolytes (sodium
phosphate, sodium chloride, sodium acetate, potassium phosphate, potassium chloride, potassium acetate, magnesium sulfate, and calcium gluconate),
multi-component vitamins and multi-component
trace minerals.

Some potential additives include cysteine, regular insulin, and additional trace vitamins or elements as required. Although omega-3 fatty acid-enriched PN formulations have been studied as a potential means of decreasing inflammation and increasing immune function in certain subsets of patients, standard PN formulations do not contain them at this time.

How well did you know this?
1
Not at all
2
3
4
5
Perfectly
68
Q

After 8 weeks at home receiving parenteral nutrition, your patient develops hair loss, a pustular rash around her mouth, and darkening of her skin creases. The most likely cause is:

A. Copper deficiency

B. Hyperkalemia

C. Hyperglycemia

D. Magnesium deficiency

E. Zinc deficiency

A

E. Because copper works with iron to form red blood cells, an early sign of copper deficiency is anemia. Low body temperature, osteoporosis, low white blood cell count, irregular heartbeat, loss of skin pigmentation, and thyroid problems may also occur due to a deficiency of copper.

Hyperkalemia is associated with a slow or irregular heartbeat and weakness.

Signs of hyperglycemia include weakness, nausea,
excessive thirst/urination/appetite, headache, irritability, and abdominal pain. Severe hyperglycemia can lead to unconsciousness.

A magnesium deficiency often manifests as cardiac and muscle irregularities, including arrhythmia, weakness, muscle cramps or spasms, restless leg syndrome, and general agitation; additional signs and symptoms of low magnesium include nausea, vomiting, insomnia, and confusion.

A pathognomonic sign of zinc deficiency is hair loss. Skin lesions, including acne, a perioral pustular and darkening of skin folds, are frequently observed. Loss of appetite, decreased motor skills, and decreased immunity may characteristic of low dietary zinc. PN must be formulated to address deficiencies at initiation as well as those that may occur over the course of hyperalimentation. While electrolyte levels are routinely monitored, one should be
aware of the potential for vitamin and trace mineral
deficits.

Individual with high output fistula or ostomy can develop metabolic disturbances. This patient may have had a zinc deficiency prior to receiving PN which should have been addressed and monitored.

How well did you know this?
1
Not at all
2
3
4
5
Perfectly
69
Q

All individuals who receive total parenteral nutrition for >13weeks will develop:

A. Venous thrombosis

B. Steatohepatitis

C. Gallbladder sludge

D. Cholelithiasis

E. Refeeding syndrome

A

C.

Catheter-related venous thrombosis is a fairly rare
complication of PN, occurring in 1% to 3% of individuals per catheter-year.

Deleterious effects of PN on the liver and gallbladder are well known to clinicians. Hepatic steatosis, which may manifest as fatty liver infiltration, may occur in PN patients within 1 to 2 weeks of initiating PN. It is reversible and can be managed by limiting the fat content. Liver function
tests (LFTs) should be checked weekly for individuals on PN and if they are elevated, lipids should be minimized to < 1 g/kd/day and total or peripheral PN should be cycled over 12 hours to rest the liver. If total bilirubin is > 5-10 mg/dL due to hepatic dysfunction, trace elements should be discontinued due to the potential for toxicity of manganese and copper.

Cholestasis is inevitable during PN because there are no intestinal nutrients to stimulate hepatic bile flow. Cholestasis typically occurs 2 to 6 weeks after starting PN and is indicated by progressive increases in total bilirubin and elevated serum alkaline phosphatase. While cholelithiasis is not uncommon during PN, it is certainly not ubiquitous.

Re-feeding syndrome is a complication that begins rapidly after starting PN in a severely malnourished individual, typically a person who has been in a starvation state for > 7-10 days. This syndrome is characterized by
a severe shift in fluid and serum electrolyte levels,
especially hypophosphatemia, resulting from intracellular electrolyte movement. Severe systemic complications, and even death, can result from re-feeding syndrome.

Correcting electrolyte abnormalities prior to initiating PN is preventative for re-feeding syndrome. Although the previous complications may occur for some individuals on PN, all PN patients will develop gallbladder sludge after receiving PN for 13 weeks.

How well did you know this?
1
Not at all
2
3
4
5
Perfectly
70
Q

If your patient on parenteral nutrition suddenly spikes a fever, the most important entity to rule out is:

A. A Crohn’s disease flare

B. A catheter line infection

C. An infection at the ostomy site

D. Clostridium difficile colitis

E. A urinary tract infection

A

B

Any of the entities listed in question 5 may cause your patient to become febrile but catheter-related bloodstream infections (CR-BSI) are the most common and most serious compications of PN.

Adequate nutrition is a cornerstone for strength preservation and immune system function in patients with serious gastrointestinal illnesses and proper training of family members and ancillary health personnel for home PN is essential.

Sterile technique when manipulating the catheter is imperative. Not all patients with a CR-BSI will present with pyrexia but a sudden increase in body temperature and an elevated C-reactive protein provide a high index of suspicion. High white blood cell count, low albumin, and/or elevated total bilirubin may be present. Catheter maintenance for home parenteral nutrition patients and repeated removal/ reinsertions can result in loss of venous access.

Whenever possible, salvage of an infected tunneled
catheter, such as the Hickman catheter used in this
patient, should be attempted. The most effective way
to prove the existence of a CR-BSI is to simultaneously draw blood cultures from the central catheter and from a peripheral source. Other sources of fever should be investigated.

How well did you know this?
1
Not at all
2
3
4
5
Perfectly
71
Q

A 27-year-old male presents with perforated appendicitis. He is in severe discomfort and shows signs of activating his sympathoadrenal axis. All of the following activate the sympathoadrenal and hypothalamic–pituitary axes during stress or injury except:

A. Pain

B. Hypovolemia

C. Acidosis

D. Hypercapnia

E. Acetylcholine

A

ANSWER: E

COMMENTS: In response to stress or injury, neural afferent signals converge on the brain to activate the sympathetic nervous system and hypothalamic stimulation. Catecholamines are released from the sympathetic nervous system and result in increases in blood pressure, heart rate, cardiac output, and minute ventilation.

Hypothalamic release of corticotropin-releasing hormone leads to release of corticotropin from the pituitary gland, which in turn induces the adrenal cortex to synthesize and release cortisol.

These responses are designed to compensate for lost circulatory volume, maintain organ perfusion, and provide the energy substrates needed for organ function. Pain is a potent activator of these pathways.

Hypovolemia simulates baroreceptors in the aorta and carotid bodies, which stimulates these pathways. Chemoreceptors in the carotid bodies and aorta are activated by hypoxemia, acidosis, and hypercapnia. These receptors also trigger the hypothalamic- pituitary-adrenal axis.

Cytokines can likewise affect these pathways, though in a less direct manner since they do not have direct neural input into these axes.

Acetylcholine has antiinflammatory effects and is not a part of the afferent response to injury.

How well did you know this?
1
Not at all
2
3
4
5
Perfectly
72
Q

The patient above exhibits signs of a systemic inflammatory response. All of the following are a part of the systemic inflammatory response syndrome (SIRS) except:

A. Temperature of 36°C or lower

B. Pulse rate lower than 56 beats/min

C. Respiratory rate of 20 breaths/min or higher

D. White blood cell count of 12,000/μL or greater

E. 10% or greater band forms on complete blood count (CBC) with differential

A

ANSWER: B

COMMENTS: The clinical spectrum of SIRS includes two or more of the following criteria:
• Temperature of 38°C or higher or 36°C or lower
• Pulse rate of 90 beats/min or greater
• Respiratory rate of 20 breaths/min or greater or a Paco2 of 32
mmHg or lower
• White blood cell count of 12,000/μL or greater or 4000/μL or lower or 10% or more band forms on the CBC with differential

SIRS is a sterile response. Sepsis includes an identifiable source of infection in addition to SIRS.

How well did you know this?
1
Not at all
2
3
4
5
Perfectly
73
Q

A patient with pheochromocytoma shows signs of an amino acid deficiency, with coarse hair, dry skin and nails, and constipation. Which of the amino acids is critical to the synthesis of catecholamines?

A. Tyrosine

B. Phenylalanine

C. Glutamate

D. Aspartic acid

E. Methionine

A

ANSWER: A

COMMENTS: Tyrosine from the diet or from conversion of phenylalanine is the prime substrate for the synthesis of catecholamines.

Tyrosine is hydroxylated to form dihydroxyphenylalanine (dopa), which undergoes decarboxylation to form dopamine.

Dopamine is then hydroxylated to form norepinephrine.

Norepinephrine is subsequently methylated in the adrenal medulla to form epinephrine.

In patients with pheochromocytoma, tyrosine can be used up and the patient can become tyrosine deficient, exhibiting the symptoms above.

How well did you know this?
1
Not at all
2
3
4
5
Perfectly
74
Q

The patient above undergoes an open adrenalectomy and has an early postoperative fever, indicating inflammation due to cytokine release. All of the following are secreted as a part of the endocrine response to stress except:

A. Corticotropin

B. Antidiuretic hormone (ADH)

C. Growth hormone

D. Thyroid hormone

E. None of the above

A

ANSWER: E

COMMENTS: Trauma induces the release of hormones, which directly affect the metabolism of carbohydrates, fats, and proteins.

Corticotropin is released from the pituitary gland and stimulates the release of cortisol, which stimulates hepatic gluconeogenesis and increases the release of amino acids from skeletal muscles.

Release of ADH from the posterior pituitary gland in response to a decreased effective circulating plasma volume leads to increased peripheral vasoconstriction, increased water reabsorption, increased hepatic gluconeogenesis, and glycogenolysis.

Growth hormone is released from the anterior pituitary and increases amino acid uptake and hepatic protein synthesis.

Thyroid hormone release increases after injury in response to the release of thyroid-stimulating hormone (TSH) from the anterior pituitary. It induces glycolysis and gluconeogenesis and increases the metabolic rate and heat production.

How well did you know this?
1
Not at all
2
3
4
5
Perfectly
75
Q

A patient presents with an aldosteronoma and clinical evidence of suppression of the renin-angiotensin system. Which of the following is true of the system?

A. It is activated by an increase in the renal tubular sodium concentration.

B. Angiotensinogen is found in the renal medulla.

C. Angiotensin-converting enzyme in the liver converts angiotensin I to angiotensin II.

D. Angiotensin II stimulates the release of aldosterone.

E. Angiotensin II decreases splanchnic vasoconstriction

A

ANSWER: D

COMMENTS: The renin–angiotensin system is activated by decreases in renal arterial blood flow and renal tubular sodium concentration, as well as increased β-adrenergic stimulation.

Renin is secreted from the juxtaglomerular cells of the renal afferent arteriole. It converts angiotensinogen in the liver to angiotensin I.

Angiotensin-converting enzyme produced by the lung converts angiotensin I to angiotensin II.

Angiotensin II simulates the release of aldosterone, increases peripheral and splanchnic vasoconstriction, and decreases the renal excretion of salt and water.

How well did you know this?
1
Not at all
2
3
4
5
Perfectly
76
Q

A 35-year-old woman presents to you after running her first marathon with complaints of muscle aches. Which of the following is not an action of cortisol in this metabolically stressed patient?

A. It stimulates release of insulin by the pancreas.

B. It induces insulin resistance in muscles and adipose tissue.

C. It stimulates release of lactate from skeletal muscle.

D. It induces release of glycerol from adipose tissue.

E. It leads to immunosuppression.

A

ANSWER: A

COMMENTS: Cortisol is the major glucocorticoid released during physiologic stress. After injury, cortisol levels are elevated in proportion to the degree of stress to the patient.

Metabolically, cortisol potentiates the actions of glucagon and epinephrine, which is manifested as hyperglycemia. It also stimulates enzymatic activities favoring hepatic gluconeogenesis.

In skeletal muscle, cortisol induces protein degradation and release of lactate; lactate serves as a substrate for hepatic gluconeogenesis.

It also potentiates the release of free fatty acids, triglycerides, and glycerol from adipose tissue to provide additional energy sources.

In a stressed patient, cortisol induces insulin resistance in muscles and adipose tissue. All these actions are directed at increasing blood glucose levels in the stressed system.

Answer A is therefore incorrect because insulin causes a decrease in blood glucose levels.

Additionally, glucocorticoids cause depressed cell-mediated immune responses (decreased killer T-cell and natural killer cell function, as well as T-cell generation) and delayed hypersensitivity responses.

How well did you know this?
1
Not at all
2
3
4
5
Perfectly
77
Q

The patient above is found to have marked rhabdomyolysis. Which of the following are effects of epinephrine in response to injury in this patient?

A. It enhances the adherence of leukocytes to vascular endothelial membranes.

B. It stimulates the release of aldosterone.

C. It inhibits the secretion of thyroid hormones.

D. It increases glucagon secretion.

E. It decreases lipolysis in adipose tissue.

A

ANSWER: D

COMMENTS: The catecholamines norepinephrine and epinephrine are increased up to fourfold in plasma immediately after injury.

In the liver, epinephrine promotes glycogenolysis, gluconeogenesis, lipolysis, and ketogenesis. It decreases insulin release and increases glucagon secretion. Epinephrine increases lipolysis in adipose tissue and induces insulin resistance in skeletal muscle. The overall effect of these actions is stress-induced hyperglycemia.

Catecholamines also increase the secretion of thyroid and parathyroid hormones as a part of the stress response.

Epinephrine induces leukocyte demargination from vascular endothelial membranes, which is manifested as leukocytosis.

How well did you know this?
1
Not at all
2
3
4
5
Perfectly
78
Q

Which of the following substances has been shown to be useful as a measurable marker of the response to injury?

A. Tumor necrosis factor-α (TNF-α)

B. Interleukin-2 (IL-2)

C. IL-6

D. IL-10

E. C-reactive protein (CRP)

A

ANSWER: E

COMMENTS: Cytokines released as a part of the stress response have a myriad of effects that both drive and inhibit the inflammatory process.

TNF-α is among the earliest detectable cytokines after injury. It is secreted by macrophages, Kupffer cells, neutrophils, natural killer cells, T lymphocytes, mast cells, and endothelial cells, among others. It has a half-life of less than 20 min.

TNF-α induces significant shock and catabolism. IL-2 is secreted by T lymphocytes and has a half-life of less than 10 min. It promotes lymphocyte proliferation, immunoglobulin production, and gut barrier integrity. It also regulates lymphocyte apoptosis.

IL-6 is released by macro- phages, B lymphocytes, neutrophils, basophils, mast cells, and endothelial cells. It has a long half-life and prolongs the survival of activated neutrophils. It is a potent inducer of acute-phase proteins in the liver.

IL-10 is secreted by B and T lymphocytes, macrophages, basophils, and mast cells. It is an antiinflammatory cytokine and has been shown to reduce mortality in animal models of sepsis and acute respiratory distress syndrome (ARDS).

CRP is useful as a marker of the response to injury because it reflects the degree of inflammation fairly accurately. CRP levels are not subject to diurnal variations and do not change with feeding. Consequently, it is used as a biomarker of inflammation and response to treatment.

How well did you know this?
1
Not at all
2
3
4
5
Perfectly
79
Q

After a gunshot wound to the lower extremity requiring operative exploration and repair of the popliteal artery, a patient has pain, pallor, and coldness of his leg. You suspect reperfusion injury causing compartment syndrome. Which of the following is true regarding reactive oxygen metabolites?

A. Reactive oxygen metabolites are synthesized and stored within leukocytes before being released in response to injury.

B. Reactive oxygen metabolites cause injury by oxidation of unsaturated fatty acids within cell membranes.

C. Cells secreting reactive oxygen metabolites are immune to damage after the release of these metabolites.

D. In ischemic tissue, the mechanisms for the production of reactive oxygen metabolites are downregulated.

E. Reactive oxygen metabolites are quenched by inhibitory cytokines.

A

ANSWER: B

COMMENTS: Reactive oxygen metabolites are short-lived, highly reactive molecules that cause tissue injury by oxidation of fatty acids within cell membranes.

They are produced during anaerobic glucose oxidation, with the resulting production of superoxide anion from the reduction of oxygen. Superoxide anion is further metabolized to hydrogen peroxide and hydroxyl radicals.

Cells are not immune to injury from the reactive oxygen metabolites that they release, but they are usually protected from damage by oxygen scavengers such as glutathione and catalases, not inhibitory cytokines.

In ischemic tissues, the mechanisms for the production of oxygen metabolites are actually activated, but because of the lack of oxygen supply, the production of reactive oxygen metabolites is kept to a minimum. Once blood flow is restored, oxygen is redelivered, thereby allowing large quantities of reactive oxygen metabolites to be produced, which in turn leads to reperfusion injury.

This is the inciting insult in compartment syndrome after the repair of vessels.

How well did you know this?
1
Not at all
2
3
4
5
Perfectly
80
Q

A patient presents with an acute gastrointestinal bleed and receives multiple transfusions of packed red blood cells. The following day, hypoxemia and bilateral infiltrates are observed on his chest x-ray. Which of the following statements about eicosanoids is true?

A. Their synthesis is dependent on the enzymatic activation of phospholipase A2.

B. They originate from lymphocytes around the site of injury.

C. They are stored within inflammatory cells and released on tissue injury.

D. The production of leukotrienes is dependent on the enzymatic activation of cyclooxygenase.

E. The production of prostaglandins is dependent on the enzymatic activation of lipoxygenase.

A

ANSWER: A

COMMENTS: Eicosanoids are a class of mediators that includes prostaglandins, thromboxanes, leukotrienes, hydroxyeicosatetraenoic acids, and lipoxins. They are secreted by all nucleated cells except for lymphocytes.

Phospholipids are converted by phospholipase A2 into arachidonic acid. Arachidonic acid is then metabolized by cyclooxy- genase to yield cyclic endoperoxides and eventually prostaglandins and thromboxanes. Alternatively, arachidonic acid is metabolized by lipoxygenase to yield hydroperoxyeicosatetraenoic acid and, eventu- ally, hydroxyeicosatetraenoic acid and leukotrienes.

Eicosanoids are not stored within cells but are synthesized and released in response to hypoxia or direct tissue injury.

Other substances such as endotoxin, norepinephrine, vasopressin, angiotensin II, bradykinin, serotonin, acetylcholine, cytokines, and histamine can also induce the production and release of eicosanoids.

Eicosanoids have a variety of deleterious effects, including acute lung injury, pancreatitis, and renal failure. They are extremely potent in promoting capillary leakage, leukocyte adherence, neutrophil activation, bronchoconstriction, and vasoconstriction.

How well did you know this?
1
Not at all
2
3
4
5
Perfectly
81
Q

In the patient above, you suspect transfusion-induced acute lung injury and intubate. The next day he has severe diffuse edema. Which of the following is true regarding the kallikrein–kinin system?

A. Bradykinins are potent vasoconstrictors produced in ischemic tissues.

B. Bradykinins are stored in macrophages and released in response to tissue injury.

C. Bradykinin release and elevation are proportional to the magnitude of injury.

D. Bradykinin antagonists have been shown to improve survival in septic trauma patients.

E. Bradykinin release is actually decreased in sepsis.

A

ANSWER: C

COMMENTS: Bradykinins are vasodilators produced by kininogen degradation by the protease kallikrein.

Kallikrein circulates in blood and tissues in an inactive form until it is activated by Hageman factor, trypsin, plasmin, factor XI, kaolin, and collagen. Bradykinins increase capillary permeability, which leads to tissue edema.

They also increase renal vasodilation, thereby leading to a reduction in renal perfusion pressure, which in turn activates the renin–angiotensin system and culminates in retention of sodium and water.

Bradykinins are released during hypoxia and ischemia and after hemorrhage, sepsis, and endotoxemia. Elevations in bradykinins are proportional to the magnitude of the injury present.

Studies in which bradykinin antagonists have been used to reduce the effects of sepsis show no improvement in survival.

How well did you know this?
1
Not at all
2
3
4
5
Perfectly
82
Q

Which of the following is true with regard to the complement cascade in the setting of injury, as in acute lung injury in the patient above?

A. Complement deactivates granulocyte activation.

B. Complement induces the release of TNF-α and IL-1.

C. Complement induces the relaxation of endothelial smooth muscle.

D. The complement components C3b and C5b are strong anaphylatoxins.

E. The complement cascade is inhibited by hemorrhage.

A

ANSWER: B

COMMENTS: Ischemia and endothelial injuries lead to the activation of complement, a series of plasma proteins involved in the inflammatory response.

Complement is activated with the release of biologically active anaphylatoxins C3a and C5a during hemorrhage. These components cause granulocyte activation and aggregation, increased vascular permeability, smooth muscle contraction, and release of histamine and arachidonic acid metabolites. They also promote the release of TNF-α and IL-1, both major cytokines in the inflammatory response.

Although the activation of complement can lead to the destruction and lysis of invading organisms, overactivation may result in tissue destruction and damage, as seen in ARDS.

How well did you know this?
1
Not at all
2
3
4
5
Perfectly
83
Q

A patient presents to the emergency room after a pitchfork puncture wound the day before, concerned about infection. Which of the following is true with regard to the inflammatory response?

A. Clot at the site of injury is the primary chemoattractant for neutrophils and monocytes.

B. Migration of neutrophils to the site of injury is inhibited by the release of serotonin.

C. Mast cells appear at the site of injury after migrating to the injury via chemoattractants such as cytokines.

D. Surgical or traumatic injury is associated with upregulation of cell-mediated immunity via type 1 helper T (TH1) cells and downregulation of antibody-mediated immunity via type 2 helper T (TH2) cells.

E. Eosinophils involved in the inflammatory response are inactivated by the complement anaphylatoxins C3a and C5a.

A

ANSWER: A

COMMENTS: Formation of clot at the site of injury serves as the primary chemoattractant for Band monocytes during the inflammatory response of the body to injury.

Migration of neutrophils along with platelets through the vascular endothelium occurs within hours of injury and is facilitated by serotonin, platelet-activating factor, and prostaglandin E2.

Mast cells are preexistent in tissues and are therefore the first to be involved in the inflammatory response. They release histamine, cytokines, eicosanoids, proteases, and TNF-α, which results in local vasodilation, capillary leakage, and recruitment of other inflammatory cells to the area.

In severe injuries, there is a reduction in cell-mediated immunity and TH1 cytokine production and a shift toward antibody-mediated immunity through the action of TH2 cells.

A TH1 response is favored in lesser injuries, with intact cell-mediated opsonizing capability and antibody immunity against microbial infections and with the activation of monocytes, B lymphocytes, and cytotoxic T lymphocytes.

A shift to the TH2 response is associated with more severe injuries and includes activation of eosinophils, mast cells, and B-lymphocyte antibody production. Eosinophils involved in the inflammatory response are activated by IL-3, granulocyte–macrophage colony-stimulating factor (GM-CSF), IL-5, platelet-activating factor, and the complement anaphylatoxins C3a and C5a.

How well did you know this?
1
Not at all
2
3
4
5
Perfectly
84
Q

In acute wounds as in the patient above, the initial recruitment of neutrophils to endothelial surfaces is mediated primarily by:

A. Immunoglobulins

B. Integrins

C. Selectins

D. All of the above

E. None of the above

A

ANSWER: C

COMMENTS: In endothelial injury, the initial recruitment of inflammatory leukocytes, specifically neutrophils, to the endothelial surfaces is mediated by adhesion molecules known as selectins, which are found on cell surfaces.

Neutrophil rolling in the first 20 min after injury is mediated by P-selectin, which is stored within endothelial cells. After 20 min, P-selectin is degraded and L-selectin becomes the primary mediator of leukocyte rolling.

Firm adhesion and transmigration of neutrophils through the endothelium and into the site of injury are mediated by integrins and the immunoglobulin family of adhesion molecules, including intercellular adhesion molecule (ICAM), vascular cell adhesion molecule (VCAM), and platelet–endothelial cell adhesion molecule (PECAM).

How well did you know this?
1
Not at all
2
3
4
5
Perfectly
85
Q

Which of the following regarding macrophages/ monocytes is true?

A. Macrophages and monocytes become hyperresponsive to continued injury/insult after trauma.

B. Functional impairment in macrophage/monocyte capability may persist for a week and is overcome with the development and growth of newer, more immature monocytes.

C. Macrophages present peptides in association with major histocompatibility complex (MHC) class II molecules to prime CD8+ cytotoxic T lymphocytes.

D. Human leukocyte antigen (HLA)/MHC II expression on monocytes increases after a major injury.

E. Macrophages present peptides in association with MHC class I molecules to prime CD4+ helper T lymphocytes.

A

ANSWER: B

COMMENTS: After the initial short-lived hyperactivation involving the release of TNF and IL-1, macrophages and monocytes actually become hyporesponsive.

Deactivation of these cells results in a type of immunologic paralysis. With stress, these cells release prostaglandin E2, which has immunosuppressive effects. It inhibits T-cell mitogenesis, along with IL-1 and TNF-α production. This functional impairment in the patient’s innate cellular immunity lasts for up to 7 days, until newly recruited monocytes are produced to bolster the immune response.

Additional mediators such as transforming growth factor-β (TGF-β), IL-10, and IL-4 are also secreted after stress or trauma and inhibit the capability of macrophages and monocytes to present antigen to T cells, thereby contributing to impairment in antigen-specific immunity as well.

The overall decrease in the adaptive immune response has been found to be associated with decreased resistance to infection. The functional impairment in macrophage/monocyte capability may persist for up to 7 days and is overcome with the development and growth of newer, more immature monocytes, which may lack the abilities of their predecessor monocytes.

HLA-DR/MHC II expression on monocytes decreases after a major injury, with prolonged depression being associated with an increased infection rate.

Macrophages present peptides in association with MHC class I molecules to prime CD8+ cytotoxic T lymphocytes and peptides in association with MHC class II to prime CD4+ helper T lymphocytes.

How well did you know this?
1
Not at all
2
3
4
5
Perfectly
86
Q

A patient with a history of alcohol abuse presents with profound hepatic failure after a Tylenol overdose. She is hypotensive, and you suspect delayed degradation of nitric oxide (NO) from her failing liver. Which of the following is true regarding NO?

A. NO is inhibited by acetylcholine stimulation.

B. NO is expressed constitutively.

C. NO can induce platelet adhesion and thus lead to microthrombosis.

D. NO has a half-life of 5 min.

E. NO is formed from the oxidation of l-alanine.

A

ANSWER: B

COMMENTS: NO is derived from the endothelial surfaces in response to acetylcholine stimulation, hypoxia, endotoxins, and cellular injury.

It is expressed constitutively at low levels and helps maintain normal vascular smooth muscle relaxation.

It reduces platelet adhesion and aggregation, thus making thrombosis of small vessels less likely.

It is diffusible, with a half-life measured in seconds.

NO is formed from the oxidation of l-arginine via the enzyme NO synthase. In liver failure, NO is not broken down efficiently, leading to hypotension.

How well did you know this?
1
Not at all
2
3
4
5
Perfectly
87
Q

A patient presents with complaints of weight loss and is found to have colon cancer. Which of the following regarding TNF-α is true?

A. It is predominantly a local mediator that induces the classic inflammatory febrile response to injury by stimulating local prostaglandin activity in the anterior hypothalamus.

B. It is effective in promoting the maturation/recruitment of functional leukocytes needed for a normal cytokine response and delays apoptosis of macrophages and neutrophils, which may contribute to organ injury.

C. It has both a proinflammatory and antiinflammatory roles; is a mediator of the hepatic acute-phase response to injury; induces neutrophil activation but also delays disposal of neutrophils; and can attenuate TNF-α and IL-1 activity, thereby curbing the inflammatory response.

D. It is an inducer of muscle catabolism and cachexia during stress by shunting available amino acids to the hepatic circulation as fuel substrates; it also activates coagulation and promotes the expression/release of adhesion mol- ecules, prostaglandin E2, platelet-activating factor, glucocorticoids, and eicosanoids.

E. It promotes T-cell proliferation, production of immuno- globulins, and gut barrier integrity.

A

ANSWER: D

COMMENTS: Cytokines are the most potent mediators of the inflammatory response. On a local level, they promote wound healing and proliferation of microorganisms.

In excess levels, as sometimes occurs during the response to injury, they may induce hemodynamic instability, which can lead to organ failure or death.

There is considerable overlap regarding the effects of cytokines with regard to promoting or attenuating the inflammatory response.

Choice A describes IL-1. 
Choice B describes GM-CSF. 
Choice C describes IL-6. 
Choice D describes TNF-α. 
Choice E describes IL-2. 

TNF-α is thought to be responsible for the cachexia observed in cancer patients.

How well did you know this?
1
Not at all
2
3
4
5
Perfectly
88
Q

Which of the following is considered an antiinflammatory
cytokine?

A. IL-1

B. IL-4

C. IL-6

D. IL-8

E. Interferon-γ (IFN-γ)

A

ANSWER: B

COMMENTS: The alterations in the hemodynamic, metabolic, and immune responses evident in stressed patients are orchestrated by endogenous polypeptides known as cytokines.

They are produced by immune cells in direct response to injury, with levels correlating with the degree of tissue damage.

Despite considerable overlap in bioactivity among cytokines, they are commonly classified by their predominant effect as proinflammatory or antiinflam- matory.

Those commonly considered proinflammatory include IL-1, IL-6, IL-8, and IFN-γ.

Those usually considered antiinflammatory include IL-4, IL-10, IL-13, and TGF-β.

How well did you know this?
1
Not at all
2
3
4
5
Perfectly
89
Q

Which of the following is true with regard to TNF-α and
IL-1?

A. Levels of soluble molecules that antagonize the effects of TNF-α and IL-1 have been shown to be predictive of organ failure.

B. Secretion of TNF-α and IL-1 is conducive to a hypoco- agulable state during an acute injury.

C. Secretion of TNF-α and IL-1 in response to injury leads
to the downregulation of the synthesis of NO and subsequent vasoconstriction.

D. TNF-α and IL-1 have a long half-life, which makes them effective markers for determining the magnitude and severity of the inflammatory response.

E. TNF-α and IL-1 have no natural antagonists; rather, their systemic effects diminish because of natural cytokine degradation.

A

ANSWER: A

COMMENTS: TNF-α and IL-1 are overproduced in patients after posttraumatic inflammation. They induce increased synthesis of NO; activation of the cyclooxygenase and lipoxygenase pathways, which leads to the formation of thromboxanes and prostaglandins; and production of platelet-activating factor, intra- cellular adhesion molecules, and selectins, which is conducive to hypercoagulability.

TNF-α and IL-1 have a short half-life, thus making them unreliable predictors of the severity of injury in the clinical setting. Soluble molecules that antagonize their effects are more stable and have been found to be predictive of lethal outcome and end-organ failure. IL-1 receptor antagonist (IL-1Ra) binds to the IL-1 receptor and blocks IL-1 activity. Soluble TNF receptors I and II (sTNF-RI and sTNF-RII) bind biologically active TNF and antagonize its effects.

How well did you know this?
1
Not at all
2
3
4
5
Perfectly
90
Q

All of the following with regard to IL-6 are true except:

A. IL-6 is a sensitive marker for the degree of tissue injury.

B. IL-6 induces the synthesis of CRP.

C. IL-6 secretion is inhibited by TNF-α and IL-1.

D. IL-6 levels peak early after injury.

E. IL-6 has antiinflammatory effects.

A

ANSWER: C

COMMENTS: IL-6 is a very sensitive marker for the degree of tissue injury. It is secreted by monocytes, macrophages, neutrophils, T and B cells, endothelial cells, smooth muscle cells, and fibroblasts. IL-6 expression is induced by bradykinin, TGF-β, platelet-derived growth factor, TNF-α, and IL-1, among others.

IL-6 levels peak early after injury, with levels found to be predictive of risk for and mortality from organ failure after trauma.

IL-6 induces the synthesis of acute-phase proteins such as fibrinogen, complement factors, α1-antitrypsin, and CRP.

CRP itself is a marker for states with increased inflammation and, in addition, is predictive of adverse outcomes following a secondary surgery. IL-6 also has some antiinflammatory effects, including inhibition of proteases and reduction of TNF-α and IL-1 synthesis; furthermore, it can cause the release of immunosuppressive glucocorticoids.

How well did you know this?
1
Not at all
2
3
4
5
Perfectly
91
Q

All of the following with regard to IL-8 are true except:

A. IL-8 levels after injury have been shown to correlate with the onset of multiorgan failure.

B. IL-8 exerts important inhibitory effects on polymorphonuclear cells.

C. IL-8 is associated with ARDS.

D. Local hypoxia induces the production of IL-8 from macrophages.

E. IL-8 does not produce the hemodynamic instability characteristic of TNF-α and IL-1.

A

ANSWER: B

COMMENTS: Like IL-6, IL-8 levels peak within the first 24 h of injury. Prolonged elevation of IL-8 is predictive of the onset of multiorgan failure and even mortality.

IL-8 is secreted by mono- cytes, macrophages, neutrophils, and endothelial cells. It is a potent chemoattractant for polymorphonuclear cells, particularly in the lung, where it is thought to have a role in initiating ARDS.

Local hypoxia is thought to play a role in stimulating IL-8 production by pulmonary macrophages. Circulating polymorphonuclear cells migrate in response to IL-8 production, thereby leading to massive infiltration into the lungs, which in turn can progress to full-blown ARDS. Interestingly, IL-8 does not produce the hemodynamic instability characteristic of TNF-α and IL-1.

How well did you know this?
1
Not at all
2
3
4
5
Perfectly
92
Q

Which of the following with regard to IL-10 is true?

A. IL-10 is a strong proinflammatory cytokine.

B. IL-10 is secreted primarily by platelets in response to injury

C. IL-10 inhibits some proinflammatory cytokines such as IL-1.

D. IL-10 has a short half-life and is therefore not a useful
marker for assessing the severity of injury.

E. IL-10 secretion is inhibited by the stress of surgical
procedures.

A

ANSWER: C

COMMENTS: IL-10 originates from T cells and monocytes. It has strong antiinflammatory properties and is capable of inhibiting the synthesis of proinflammatory cytokines such as IL-1 and TNF- α. It also induces a reduction in class II MHC molecules on monocytes, thereby leading to the downregulation of the immune response.

IL-10 levels in trauma patients have been shown to reflect the severity of injury and are predictive of patients in whom sepsis or multiorgan dysfunction syndrome will develop.

Release of IL-10 is increased in direct proportion to tissue damage, thus suggesting that more invasive surgical procedures augment the release of IL-10.

How well did you know this?
1
Not at all
2
3
4
5
Perfectly
93
Q

A patient in your intensive care unit (ICU) has hyperglycemia at 250 mg/dL 17 h after coronary artery bypass grafting. All of the following regarding insulin therapy are true except:

A. Hyperglycemia increases the morbidity of critically ill patients in the surgical ICU setting without significantly affecting mortality rates.

B. Maintaining blood glucose levels of 80 to 110 mg/dL is beneficial in surgical ICU patients.

C. Hyperglycemia impairs macrophage ability.

D. Insulin has antiinflammatory effects.
E. Hyperglycemia promotes coagulation.

A

ANSWER: A

COMMENTS: Prospective, randomized data from Van den Berge and colleagues have shown that hyperglycemia increases mortality rates in critically ill surgical ICU patients.

Hyperglycemia promotes oxidative stress, coagulation, and phagocyte dysfunction. Advanced glycation end products resulting from hyperglycemia are them- selves proinflammatory.

Insulin has anabolic, antiinflammatory, and antiapoptotic effects.

For all these reasons, insulin therapy for tight blood glucose control has been shown to improve outcomes in ICU patients.

However, maintaining very strict glucose control (80 to 110 mg/dL) has been shown to worsen outcomes and increase mortality, and a higher threshold is recommended.

How well did you know this?
1
Not at all
2
3
4
5
Perfectly
94
Q

The above patient remains intubated and is without enteral feeding for 3 days. Which of the following is the main energy source during critical illness/injury?

A. Skeletal muscle

B. Liver

C. Adipose tissue

D. Kidney

E. Gut

A

ANSWER: C

COMMENTS: Lipids are nonprotein, noncarbohydrate fuel sources that minimize protein breakdown in injured patients.

In response to catecholamines released during stress, triglyceride lipase induces fat mobilization/lipolysis from adipose stores. Glycerol is released and provides a substrate for hepatic gluconeogenesis.

Fatty acids are released and processed into ketone bodies by the liver to provide an additional fuel source. Free fatty acids can also serve as a direct source of energy for such tissues as cardiac, kidney, liver, and muscle cells.

How well did you know this?
1
Not at all
2
3
4
5
Perfectly
95
Q

When attempting to wean from the ventilator on postopera- tive day 6, you notice the above patient has persistent hypercapnea and suspect overfeeding. Which of the follow- ing is correct with respect to the respiratory quotient (RQ)?

A. RQ = 1: greater oxidation of protein for fuel

B. RQ > 1: overfeeding/greater carbohydrate oxidation

C. RQ = 0.7: greater oxidation of carbohydrate for fuel

D. RQ = 0.85: greater oxidation of fatty acid for fuel

E. RQ < 1: excess breakdown of proteins for fuel

A

ANSWER: B

COMMENTS: The RQ is a unitless number used for the calcula- tion of basal metabolic rate when estimated from carbon dioxide production. It is calculated from the ratio of CO2 produced and O2 consumed.

The RQ in patients who are in metabolic balance usually ranges from 1.0 (the value expected for pure carbohydrate oxidation) to 0.7 (the value expected for pure fat oxidation).

A mixed diet of fat and carbohydrate results in an average value between these numbers. The RQ may rise above 1.0 in an organism oxidizing carbohydrate to produce fat, as in overfeeding.

In summary,
RQ = 1: greater oxidation of carbohydrate for fuel
RQ > 1: overfeeding/greater carbohydrate oxidation
RQ = 0.7: greater oxidation of fatty acid for fuel
RQ = 0.85: oxidation of equal amounts of fatty acids and glucose

96
Q

For an adult patient consuming a normal diet, which of the following is the most calorically dense energy source?

A. Fat

B. Alcohol

C. Protein

D. Carbohydrate E. Water

A

ANSWER: A

COMMENTS: Fat stores can equal 160,000 kcal, with higher stores present in obese individuals. Lean body mass proteins can supply 30,000 kcal of the body’s energy stores. Energy in the diet can be provided by carbohydrates (4 kcal/g), fats (9 kcal/g), pro- teins (4 kcal/g), and alcohol (7 kcal/g).

97
Q

A 53-year-old diabetic patient undergoes small bowel resection for volvulus. He now has a prolonged postoperative ileus and has had only 0.45% normal saline for 5 days. In order to limit protein catabolism, how much glucose should be administered in his total parenteral nutrition (TPN)?

A. 1 L of 5% dextrose in water (D5W)

B. 2 L of D5W

C. 3 L of D5W

D. 4 L of D5W

E. 5 L of D5W

A

ANSWER: B

COMMENTS: Blood glucose levels are regulated by hormones in response to carbohydrate intake. Insulin secretion increases with intake of glucose, and glucagon secretion declines, thus allowing an increased uptake of glucose by liver, muscle, and adipose tissue.

Conversely, glucagon mobilizes liver glycogen via the cyclic adenosine monophosphate (cAMP) protein kinase system when blood glucose levels decrease because of decreased intake.

Glucose tolerance is determined by the rate at which mechanisms of glucose removal can operate.

Administration of 100 g of glucose (or 1 mg/ kg/min) has a protein-sparing effect that suppresses the use of nitrogen (from amino acids) for gluconeogenesis. D5W, or 5% dextrose per liter of water, contains 50 g of dextrose per liter.

98
Q

A patient with a history of a trauma laparotomy presents with a small bowel obstruction. A nasogastric tube is placed that aspirates greater than 4L of fluid per day. The gastrointestinal tract can secrete and reabsorb how much water in the form of gastric juices per day (in a 70-kg adult male)?

A. 1 to 2 L/day

B. 4 to 5 L/day

C. 6 to 7 L/day

D. 8 to 10 L/day E. 50 L/day

A

ANSWER: D

COMMENTS: The end products of protein, carbohydrate, and fat oxidation include water, with 1 g of carbohydrate yielding 0.6 mL of water, 1 g of protein yielding 0.42 mL, and 1 g of fat yielding 1.07 mL.

In addition, the gastrointestinal tract may secrete and reabsorb as much as 8 to 10 L/day of water as digestive juices in the following estimated amounts: saliva, 1500 mL; gastric juice, 2500 mL; bile, 500 mL; pancreatic juices, 700 mL; intestinal juices, 3000 mL; and water intake, 2000 mL.

99
Q

In the above patient, the decreased insulin–glucagon ratio seen during simple starvation allows:

A. Increased lipogenesis

B. Increased lipolysis

C. Increased protein synthesis

D. Increased glycogen production

E. Decreased lipolysis

A

ANSWER: B

COMMENTS: During starvation, the insulin–glucagon ratio is decreased, which allows activation of lipolysis and suppression of lipogenesis.

Lipolysis breaks down fat to free fatty acids, which are used as alternative fuels by many tissues that prefer fat as a fuel source (i.e., kidney, cardiac muscle, and skeletal muscle).

The liver oxidizes fatty acids to acetyl CoA, which is then converted to ketones. As ketone (acetoacetate and β-hydroxybutyrate) levels rise, they can cross the blood–brain barrier to supply fuel, but some glucose is still required.

The use of fatty acids as a primary fuel source allows the sparing of body proteins for gluconeogenesis. This sparing effect is important for maintenance of immune functions and liver and respiratory muscle function. A respiratory quotient (RQ, Vco2/Vo2) of 0.6 to 0.7 during simple starvation reflects the fact that fat is the body’s primary fuel source during simple starvation.

100
Q

During simple starvation, gluconeogenesis is important for:

A. Glycogen storage

B. Lipogenesis to continue to allow adequate fat storage

C. Protein synthesis to progress to allow muscle health

D. Tissues that use only glucose for fuel, such as the
brain and blood, and are dependent on this process

E. None of the above

A

ANSWER: D

COMMENTS: Simple starvation results when nutrient intake does not meet energy requirements. Energy expenditure characteristically decreases to help match energy intake, and metabolic responses occur to preserve the muscle mass.

In early fasting, glycogenolysis is the first energy source, followed by gluconeogenesis to maintain a glucose supply to obligate glucose users such as the brain, blood, renal medulla, and bone marrow.

Gluconeogenesis requires alanine, glycerol, and lactate; therefore the focus of other organ systems in early fasting is to provide these substrates to the liver.
Initially, during early fasting, the glycogen derived from glycogenolysis supplies glucose for obligatory glucose-using tissues.

Lipogenesis is curtailed. The Cori cycle is then activated, which allows glucose to be converted back to lactate through glycolysis in the peripheral tissues. Skeletal muscle releases alanine via the alanine cycle. Through these reactions, alanine and lactate are created for gluconeogenesis.

101
Q

Which amino acid is released in large amounts to be used by the liver during simple starvation?

A. Valine

B. Serine

C. Glutamine

D. Cysteine

E. Homocysteine

A

ANSWER: C

COMMENTS: Protein metabolism adapts to starvation as follows:

(1) the synthesis of protein decreases because energy sources for production are not available,
(2) protein catabolism is reduced as other fuels become the primary sources of energy for many tissues, and
(3) decreased ureagenesis and urinary nitrogen loss reflect protein sparing (in the initial stages of starvation, the rate of urea nitrogen loss is greater than 10 g/day, with a decline to less than 7 g/day after weeks of starvation).

Alanine, glutamine, and glycine are released in large amounts to be used by the liver and kidney for net glucose formation.

102
Q

A 24-year-old male is made nil per os (NPO) after midnight for lipoma removal the next day. His case is delayed until early evening, and he enters simple starvation. In this patient, glucagon mobilizes which of the following?

A. Glycogen from muscle tissue

B. Liver glycogen

C. Insulin to improve the cellular uptake of glucose

D. Glucose to the liver for storage

E. None of the above

A

ANSWER: A
COMMENTS: Carbohydrates can be classified as complex (poly- saccharides) or simple (mono- and disaccharides). Carbohydrate digestion starts in the mouth with salivary amylase, which hydro- lyzes polysaccharide bonds, and continues with pancreatic amylase and the enzymes sucrase, lactase, maltase, and isomaltase from intestinal epithelial cells to yield monosaccharides.
Glucose, a monosaccharide, is the preferred fuel in humans, with all the metabolism beginning or ending with this hexose. Glucose is transported to the liver via the portal circulation and goes on to form pyruvate, glycogen, or fat in adipose tissue.
Glycogen is the stored form of carbohydrates and is present in the liver and the skeletal muscle; the liver glycogen is for general use and can be mobilized at any time using the enzyme glucose-6-phosphatase, whereas muscles lack this enzyme and therefore conserve their glycogen for exclusive muscle use.
In times of fasting, glucagon activates the enzyme glycogen phosphorylase to facilitate glycogenolysis. Glucagon also inhibits glycogen synthase, preventing the formation of new glycogen. Gly- cogenolysis can provide glucose supplies for 12 to 24 h of fasting.

103
Q

A patient presents with lymphoma and a left pleural effusion; upon drainage, the fluid is noted to be chylous. In which of the following is he likely to be deficient?
A. Short-chain fatty acids
B. Medium-chain fatty acids C. Long-chain fatty acids
D. Calcium
E. Vitamin B12

A

ANSWER: C
COMMENTS: Fat absorption is through the small intestine and the lymphatics.
Medium- and short-chain free fatty acids enter the entero- cyte through simple diffusion and are transported from there into the portal system. Triacylglycerides (TAGs), cholesterol, and lipids are broken down by pancreatic lipase, cholesterol esterase, phospholipase, and bile salts to form micelles and free fatty acids in the intestine. Micelles contain bile salts, long-chain fatty acids, and cholesterol. The micelles then fuse with the enterocyte mem- brane and from there undergo breakdown in the enterocyte.
TAGs are then resynthesized and form chylomicrons with phospholipids and cholesterol. These, along with long-chain fatty acids, enter the lymphatics to empty into the thoracic duct.
Chylothorax and other lymphatic or thoracic duct disruptions can result in a profound deficiency of TAGs and long-chain fatty acids.

104
Q

A patient presents with a 55% total body surface area (TBSA) burn. Early enteral feeds are started per protocol, and the burn unit director considers glutamine supplementa- tion. Glutamine is an amino acid that:
A. Is categorized as an essential amino acid
B. Is found only in muscle tissue
C. Has been shown to be conditionally essential during stress
D. Maintains stable levels in plasma during stress
E. Can be eliminated from the diet during times of stress

A

ANSWER: C
COMMENTS: Body proteins are made up of 20 different amino acids, each of which has a different metabolic rate and function in the body. There are three categories of amino acids: (1) essential amino acids, which cannot be synthesized by the body; (2) nones- sential amino acids, which can be synthesized de novo in the body; and (3) conditionally essential amino acids, which consist of nones- sential amino acids that are considered essential during stress or trauma if their use exceeds the body’s capacity for synthesis and an outside source is required.
Glutamine, the most abundant amino acid in the body, is a conditionally essential amino acid; it accounts for 50% of the amino acids in muscle, and its concentrations can fall during times of stress because of the body’s inability to meet increases in body requirements for the amino acid. Glutamine is the major amino acid for intestinal mucosa, and some evidence shows improved out- comes in burn patients with glutamine supplementation. Other con- ditionally essential amino acids include arginine and tyrosine.

105
Q

Which amino acids are classified as essential, can be metabolized outside the liver, and are a local source of energy for muscle?
A. Leucine, isoleucine, and valine
B. Alanine, arginine, and lysine
C. Ethionine, glutamine, and lysine
D. Phenylalanine, tyrosine, and histidine E. None of the above

A

ANSWER: A
COMMENTS: Essential amino acids are those that cannot be synthesized and must be obtained in the diet. The essential amino acids are valine, leucine, isoleucine, methionine, lysine, threonine, phenylalanine, and tryptophan. Of these, valine, leucine, and iso- leucine are branched-chain amino acids; this makes them less sus- ceptible to first-pass metabolism and less amenable to breakdown in the liver. They are easily broken down by muscle and therefore provide an energy source to skeletal muscle. The breakdown prod- ucts of these amino acids are alanine and glutamine.

106
Q

A 65-year-old woman with recently diagnosed nonobstructing colon cancer presents to schedule a right hemicolectomy. She has read about nutrition and healing after her surgery and asks how she can optimize her protein intake. What are the dietary protein recommendations for a 60-kg woman with intact protein stores?

A. 0.7 to 0.8 g/kg/day (42 to 48 g/day)

B. 0.9 to 1.0 g/kg/day (54 to 60 g/day)

C. 1.2 to 1.5 g/kg/day (72 to 90 g/day)

D. 2 to 4 g/kg/day (120 to 240 g/day)

E. 5 to 6 g/kg/day (300 to 360 g/day)

A

ANSWER: A

COMMENTS: The dietary protein requirement for adults is 0.8 g/ kg/day; that is, approximately 20% of the calories consumed should be in the form of protein.

One gram of nitrogen equals 6.24 g of protein. In patients undergoing acute stress such as that of surgery, protein intake should rise to 1.2 to 1.5 g/kg/day; in patients with severe stress, such as burns or large nonhealing wounds, protein requirements may rise to 2 to 4 g/kg/day. For burn patients, protein requirements may be calculated by the following formula:
Protein = 1.5 g/kg/day + (3 g × % TBSA burn)

107
Q

A patient presents with a perforated Meckel’s diverticulum and undergoes terminal ileal resection with reanastomosis. He subsequently develops poor reabsorption of bile acids. What is the primary substrate for the formation of bile acids?

A. Cholesterol

B. Triglycerol

C. Triglycerides

D. Phospholipids

E. Insulin

A

ANSWER: A

COMMENTS: Three main forms of fat are found in the body: glycerides, phospholipids, and sterols.

Glycerides, principally triglycerides and triglycerol (fatty acid and glycerol), are the storage forms of fat and are the most abundant forms in food; they account for approximately 95%–98% of ingested fat and the fat in tissues.

Triglycerides store calories, protect organs, and act as insulators.

Phospholipids are ingested in small amounts and are main constituents of cell membranes and myelin sheaths; they are the substrates for prostaglandins, leukotrienes, and thromboxanes.

Sterols consist primarily of cholesterol. Cholesterol is the substrate for the formation of bile acids (the primary bile acids are cholate and chenodeoxycholate) and steroid hormones (aldosterone, progesterone, estrogen, and androgens).

108
Q

A 26-year-old patient presents with abdominal pain, weight loss, and steatorrhea. He is found to have severe terminal ileal involvement and structuring. In which of the following vitamins is he least likely to be deficient?

A. Vitamin A

B. Vitamin D

C. Vitamin E

D. Vitamin C

E. Vitamin K

A

ANSWER: D

COMMENTS: Vitamins can be either water soluble (vitamin C and B vitamins—thiamin, niacin, riboflavin, folate, vitamin B6, vitamin B12, biotin, and pantothenic acid) or fat soluble (vitamins A, D, E, and K). Patients with steatorrhea, as seen in those with an absent or damaged terminal ileum or those with pancreatitis, can present with deficiencies of the fat-soluble vitamins.

The following are the classic signs of vitamin deficiency:
Vitamin A—dermatitis, night blindness, and poor wound healing Vitamin D—bone demineralization and osteopenia

Vitamin E—increased platelet aggregation and decreased red blood cell survival

Vitamin K—bruising and hemorrhage

109
Q

Which of the following is not a component of the Harris–
Benedict equation?

A. Weight

B. Height

C. Age

D. Gender

E. % Lean body mass

A

ANSWER: E
COMMENTS: The Harris–Benedict equation can be used to predict the basal metabolic rate of a given patient at rest. The equation factors in weight, height, age, and gender form an estimation of caloric needs. It is not an infallible estimate but is commonly used as baseline estimation.

In general, a patient’s diet should be broken into 20% protein, 30% fat, and 50% carbohydrates. Protein needs may increase in the postsurgical state or when significant wound healing is necessary, as in burns.

Trauma, surgery, sepsis, pregnancy, lactation, and fever can increase basal metabolic needs significantly.

110
Q

A 75-year-old female presents with ductal carcinoma in situ for simple mastectomy. She has marked muscle wasting on examination and admits to a poor diet.

Which of the following values is most predictive of postoperative mortality?

A. Serum sodium

B. Serum albumin

C. Serum protein

D. Serum creatinine

E. Serum glucose

A

ANSWER: B

COMMENTS: Nutritional assessment of surgical patients involves evaluation for preexisting malnutrition, comorbidities, risk factors for malabsorption, and substance dependency.

A number of laboratory tests may be performed based on the patient’s individual risk factors. Multiple test values and anthropomorphic measurements have been used to evaluate the risk of nonhealing and complications after surgery; however, only serum albumin has been found to correlate directly with morbidity and mortality.

Preoperative albumin levels less than 3 g/dL are associated with an increased risk of developing serious complications within 30 days of surgery, including sepsis, acute renal failure, coma, failure to wean from ventilation, cardiac arrest, pneumonia, and wound infection. A serum albumin level greater than 4.5 carries a <1% risk of mortal- ity, less than 3 carries a 9% risk, and less than 2 carries a 30% risk.

111
Q

A 55-year-old woman presents with pneumoperitoneum and is found to have a perforated gastric ulcer with gross intraabdominal spillage. Subsequently, she has sepsis and a prolonged intubation, and tube feeds are started. Which of the following visceral proteins is the best indicator of her immediate nutritional status?

A. Prealbumin

B. Albumin

C. Transferrin

D. Total protein

E. Serum globulin

A

ANSWER: A
COMMENTS: In patients with a prolonged lack of nutrition, serum markers of nutrition are a helpful way of assessing the adequacy of nutritional supplementation.

Serum total protein levels are nonspecific and can be elevated by viral or autoimmune disease processes.

Levels of serum albumin, which has a half-life of 18 to 21 days, are helpful in the outpatient setting to assess general nutrition but are poorly reflective of short-term changes while hospitalized.

Serum transferrin is commonly used but has a half-life of 8 to 10 days, again reflecting too long a time period to provide day- by-day assessments.

Serum prealbumin has a half-life of 10 h to 2 days, depending on which isotype is used, and is commonly obtained to assess the response to supplementation.

Of note, all of these markers are hepatic acute-phase reactants and can decrease in times of stress; therefore the trend is more significant than the absolute value.

112
Q

A patient presents to the trauma bay after a house fire with a 65% TBSA burn. He is intubated and remains in the burn unit for 5 days, after which he enters the hypermetabolic flow phase of stress metabolism.

Hyperglycemia during stress hypermetabolism can be attributed to:

A. Increased insulin resistance

B. Increased glycogen storage

C. Decreased lipolysis

D. Increased glycogenesis

E. Increased insulin uptake

A

ANSWER: A

COMMENTS: In contrast to simple starvation, activation of stress hypermetabolism occurs following surgery, trauma, or sepsis to provide energy and substrates for tissue repair and to activate immune function and the inflammatory response.

In the initial period, known as the ebb phase, a decline in oxygen consumption is seen, along with poor circulation, fluid imbalance, and cellular shock lasting 24 to 36 h.

As the body adapts (flow or plateau phase), enhanced cellular activity and increased hormonal stimulation take place and lead to an elevated metabolic rate, body temperature, and nitrogen loss. This phase can last days, weeks, or months.

Both phases are characterized by hyperglycemia. In the ebb phase, patients manifest impaired glucose tolerance. In the flow phase, patients display frank insulin resistance.

In the hypermetabolic phase, glycogen storage is attenuated and lipogenolysis is increased in an effort to keep up with the grossly increased metabolic demands of wound healing.

113
Q

In the patient above, stress hypermetabolism is characterized by:

A. Decreased body temperature

B. Hypoglycemia and glycogenesis

C. Fluid imbalance and increased resting metabolic rate

D. Decreased gluconeogenesis and proteolysis

E. Decreased urinary protein retention

A

ANSWER: C

COMMENTS: The earliest stages of hypermetabolism are characterized by increases in gluconeogenesis, resting energy expenditure (REE), proteolysis, ureagenesis, and urinary nitrogen loss.

Clinical signs include tachypnea, increased body temperature, and tachycardia, with laboratory results showing increased leukocytosis, hyperlactatemia, azotemia, and hyperglycemia.

Liver production of glucose during stress is increased through gluconeogenesis and glycogenolysis (Cori cycle), which are stimulated by endocrine (hormonal) changes: increased cortisol, increased glucagon, increased catecholamines, and decreased insulin.

Overall use of protein as an oxidative fuel source by the liver is increased, and typically, there is increased turnover of branched-chain amino acids.

114
Q

A 59-year-old woman remains intubated due to sepsis after perforated diverticulitis. She has failed multiple trials of extubation due to hypercapnia, which you suspect to be due to overfeeding. Which of the following RQs is characteristic of overfeeding?

A. 0.75

B. 0.85

C. 0.90

D. 1.0

E. 1.05

A

ANSWER: E
COMMENTS: Calculation of the RQ allows the clinician to alter the nutrient content of feedings to optimize the macronutrient intake.

The RQ is the ratio of CO2 expired (Vco2) to the amount of O2 inspired (Vo2): RQ = Vco2/Vo2.

The RQ is reflective of the type of energy expenditure. CO2 is produced when carbohydrates are converted to fat; therefore an elevated RQ (1.0) indicates pure carbohydrate metabolism, and an RQ of >1.0 indicates conversion of carbohydrates into fat and therefore overfeeding.

An RQ of 0.7 is consistent with pure fat metabolism (starvation), 0.8 with pure protein metabolism, and 0.83 with a balanced metabolism. A high RQ, by definition, causes hypercapnia and can lead to difficulty to wean from the ventilator.

115
Q

A 35-year-old man is admitted to the intensive care unit (ICU) following a diagnosis of acute pancreatitis. After initial resuscitation, the patient’s condition improves and enteral tube feedings are started through a postpyloric tube. Initial intolerance to a tube feeding regimen requires the clinician to:

A. Immediately start TPN

B. Add water to the feeding regimen to dilute the feedings for better tolerance

C. Slow the tube feeding regimen and progress to the goal rate less aggressively

D. Immediately change the tube feeding formula

E. Increase the tube feeding rate per hour

A

ANSWER: C

COMMENTS: Enteral feedings have been shown to decrease septic complications and are theorized to prevent intestinal bacterial translocation. Complications can be metabolic (e.g., overhydration or underhydration) or gastrointestinal (e.g., diarrhea, nausea, vomiting, delayed gastric emptying, constipation, or abdominal distention).

Diarrhea has been estimated to occur in 2.3% of the enteral population and in 34%–41% of critically ill patients. Diarrhea may be related to (1) factors associated with the patient’s underlying illness or (2) factors related to the tube feeding formula, including too rapid an infusion rate, too rapid initiation or progression, lactose intolerance, microbe contamination, lack of fiber, the osmolality of the formula, or a high fat content in the formula.

The diarrhea may be controlled by (1) antimotility agents, (2) changing to continuous feeding, or (3) slowing the rate of tube feeding until tolerance is established, before initiation of medication to control the problem.

116
Q

A 67-year-old woman with a history of atrial fibrillation is admitted to the emergency department with complaints of abdominal pain out of proportion to the physical findings. She is found to have superior mesenteric arterial occlusion and undergoes on-table thrombectomy and stenting with small bowel resection. The remaining proximal jejunum measures 100 cm. What is the minimum amount of small intestine required for absorption of nutrients before consider- ing the use of enteral feedings?

A. 20 cm of small intestine

B. 50 cm of small intestine

C. 100 cm of small intestine

D. 120 cm of small intestine

E. 250 cm of small intestine

A

ANSWER: C

COMMENTS: The functional capacity of the gut must be considered before prescribing enteral nutrition therapy. There must be at least 100 cm of the small intestine for absorption of nutrients with an intact ileocecal valve, or 150 cm with an incompetent ileocecal valve.

Relative contraindications for the use of the gastrointestinal tract include gastroparesis, intestinal obstruction, and paralytic ileus. More absolute contraindications include high-output enteric fistula, short bowel syndrome, severe malabsorption, and hemody- namic instability.

117
Q

Due to multiple takebacks for “second-look” procedures and attempts at extubation, the patient in the question above remains without enteral nutrition for 8 days. Which type of formula might be appropriate for a patient who has been NPO for more than 1 week and has a partially functioning gastrointestinal tract?

A. Elemental formula

B. Concentrated formula

C. Specialty formula

D. Modular formula

E. Superconcentrated formula

A

ANSWER: A
COMMENTS: Tube feeding formulas are divided into six types: standard, concentrated, high nitrogen, elemental, fiber containing or blenderized, and special. They differ by nutritional content and texture.

Standard formulas contain 50%–60% of calories being derived from carbohydrates, 10%–15% from protein, and 25%–40% from fat. They are appropriate for patients with no major enteral restrictions who have been NPO for less than 1 week.

Concentrated formulas have similar nutritional content but decreased water content, making them suitable for patients with fluid restrictions or those receiving bolus feedings.

Elemental formulas contain hydrolyzed macronutrients, requiring less energy and mucosal interface to digest; they are appropriate for those who have had a prolonged NPO status or those with tube feed intolerance. High- nitrogen-protein formulas provide more than 15% of calories from proteins and are appropriate for those with high protein needs.

Fiber-containing and blenderized formulas contain increased fiber to decrease diarrhea. Special formulas have special formulations for special patient populations, such as those with renal failure or hepatic failure.

118
Q

Which of the following is one of the most common food allergies that must be considered when deciding on a tube feeding formula?

A. Rice allergy

B. Soy allergy

C. Nut allergy

D. Corn syrup allergy

E. Citrus fruit allergy

A

ANSWER: C

COMMENTS: Food allergy considerations are important when selecting an enteral formula. Nuts, fruits, and milk are the most common food allergy triggers for 20% of the population of allergy sufferers. Gluten tolerance should also be a consideration. Some enteral products contain lactose, which causes bloating, cramping, and diarrhea in some patients.

Because of electrolyte imbalances or the need for varied macronutrient content, some patients have needs that cannot be met by the available commercial products.

Enteral feeding modules can be used to create a patient-specific formula (modular formula) that addresses the carbohydrate, protein, and fat requirements.

119
Q

How many calories are provided in one 500-mL bottle of
20% intravenous fat solution?

A. 150 kcal

B. 550 kcal

C. 800 kcal

D. 1000 kcal

E. 4000 kcal

A

ANSWER: D

COMMENTS: Fat solutions are available in 10%, 20%, and 30% solutions, which provide 1.1, 2.0, and 3.0 kcal/mL of energy, respectively. In this question, 500 mL of a 20% infusion provides 1000 kcal.

120
Q

A 65-year-old man is admitted to the hospital because of profuse diarrhea after small bowel resection for an ischemic bowel that resulted in short bowel syndrome. The patient is resuscitated and TPN started. What is the maximum infusion rate for lipids when using TPN?

A. 0.5 g/kg/day

B. 1.5 g/kg/day

C. 2.5 g/kg/day

D. 3.0 g/kg/day

E. 4.0 g/kg/day

A

ANSWER: C

COMMENTS: A patient whose nutritional needs cannot be met via the oral or enteral route requires TPN, the basic goal of which is to meet nutritional needs and maintain or improve metabolic balance safely.

Crystalline protein and synthetic amino acids are the protein sources for TPN solutions. Commercially available dextrose solutions typically contain 15%–35% dextrose.

The monohydrate form of dextrose is used for TPN and provides 3.4 kcal/g on oxidation.

Carbohydrates should be administered at a rate no greater than 5 mg/ kg/min via TPN, which is the maximum oxidation rate of glucose.

Fat solutions are considered isotonic and can be administered peripherally or centrally without concern about thrombosis. Fat solutions are available in 10%, 20%, and 30% solutions, which provide 1.1, 2.0, and 3.0 kcal/mL, respectively. Monitoring clearance by checking triglyceride levels is key to ensuring tolerance to lipid infusions. Fat should be administered at a rate no faster than 2.5 g/kg/ day. Lipid solutions should be administered cautiously to patients with acute respiratory distress syndrome (ARDS), severe liver disease, or increased metabolic stress because fat may exacerbate these conditions.

Patients with hypertriglyceridemia (>250 mg/dL), lipid nephrosis, egg allergy, or acute pancreatitis associated with hyperlipidemia should not be given fat emulsions.

121
Q

A patient with multiple small bowel resections for Crohn’s disease presents with an acute flare. Due to his preexisting short length of bowel and current profuse diarrhea, he is suspected of having functionally a short gut syndrome and is started on TPN. Refeeding syndrome is characterized by which of the following electrolyte abnormalities?

A. Hyponatremia, hypokalemia, and hypercalcemia

B. Hyperphosphatemia, hypokalemia, and hypocalcemia

C. Hypokalemia, hypomagnesemia, and hypophosphatemia

D. Hypocalcemia, hyponatremia, and hypomagnesemia

E. Hyperkalemia, hypernatremia, and hypercalcemia

A

ANSWER: C

COMMENTS: Refeeding syndrome is a profound metabolic disturbance that can occur in patients who are in severe malnutrition prior to starting TPN.

Initiation of TPN can result in markedly decreased serum potassium, magnesium, and phosphorus; this is due to the effects of glucose administration in the previously starved patient.

Refeeding can be prevented by evaluating and repleting serum levels of these electrolytes prior to starting TPN.

122
Q

After 4 days of TPN, the patient above develops blood glucose levels greater than 300 mg/dL. Hyperglycemia in a surgical patient receiving TPN may best be managed by:

A. Oral hypoglycemics

B. Decreasing the dextrose load and doubling the amount of fat

C. Adding regular insulin to the TPN

D. Discontinuing TPN for 2 weeks and then trying to start TPN again

E. Increasing the concentration of protein and carbohydrate calories and decreasing that of lipids

A

ANSWER: C

COMMENTS: A number of metabolic complications may result from TPN.

Excess glucose can (1) increase blood glucose levels and induce hyperosmolar nonketotic coma; (2) lead to dehydration; (3) lead to lipogenesis with subsequent hepatic abnormalities (e.g., fatty liver); and (4) increase CO2 production, which may compromise respiratory function.

Because rebound hypoglycemia can occur with discontinuation of TPN, weaning to 50 mL/h before complete discontinuation is important.

Treatment of hyperglycemia in TPN patients typically consists of the addition of regular (not long-acting) insulin to the parenteral solution along with stringent monitoring of glucose levels.

123
Q

A 40-year-old man undergoes gastric bypass surgery for morbid obesity. This patient should:

A. Eat only high-protein foods

B. Begin a feeding regimen with small amounts of regular foods

C. Begin with small amounts of water

D. Eat high-calorie foods six times a day

E. Eat only small amounts of high-fat foods

A

ANSWER: C

COMMENTS: Bariatric surgery has emerged as a viable weight-loss method for patients with medically significant obesity and morbidly obese patients who are unable to achieve or sustain weight loss.

Nutritional issues are complex in this population. Patients are expected to lose 50%–60% of their presurgery weight. Therefore numerous postoperative nutritional complications are common. Malnutrition, vitamin and mineral deficiencies, failure of weight loss, dehydration, anemia, and dumping syndrome may occur.

Oral feeding resumes on postoperative day 1 with small volumes of water. A 3-day progression from clear liquids to pureed foods is recommended, with small-volume feedings of 30 to 60 mL at each feeding.

The diet eventually returns to regular foods given in small, frequent meals, along with the following general instruc- tions: (1) stop eating when full; (2) chew food well, to a pulp-like consistency; (3) avoid high-calorie liquids, especially those with ice cream; and (4) make mealtimes last 30 min.

Because of bypass of 90% of the stomach, entire duodenum, and a small portion of the jejunum, supplemental nutrient recommendations are necessary. B6, B12, folate, and calcium deficiencies are common, and these nutrients are usually prescribed as supplements.

124
Q

Which of the following is true when considering the
nutritional status of a geriatric patient?

A. Muscle wasting can be a pathologic process that is mistaken for normal aging.

B. Liver function does not affect the selection of nutrition regimens.

C. Enteral nutrition is not an option because of slowed gut function.

D. Body mass index (BMI) is the best anthropometric measurement for determining the nutritional status in an elderly patient.

E. Laboratory tests cannot be used to evaluate the nutritional status.

A

ANSWER: A

COMMENTS: Determining the most appropriate nutrition support interventions in elderly surgical patients can be difficult because of the aging process. Use of the BMI and anthropometrics often results in inaccurate measurements because of age-related changes. Many pathologic processes mistaken for normal aging are related to nutrition or affect the nutritional status (anthropometrics and biochemical and hematologic aspects) and can include muscle wasting, weight loss, undernutrition, problems with balance and endurance, declining cognition, and depression.

Multiple age- related changes in the renal, liver, cardiovascular, and muscular systems can affect the overall health and nutrition regimens for surgical patients.

Enteral nutrition may be an option for an elderly patient with poor nutritional intake before surgery. Commonly, enteral nutrition has been used in patients with dementia, cancer, dysphagia secondary to stroke, and other neurologic problems.

Conditions for which the benefits of enteral support have been demonstrated in the elderly are short-term use in those with stroke and cancer when significant decreases in body mass can probably be prevented, the patient is a candidate for physical therapy, or recovery is probable.

125
Q

Mnemonic for the composition of body fluid:

A

60, 40, 20;
60% total body weight fluid,
40% total body weight intracellular,
20% total body weight extracellular

126
Q

Approximate percentage of body weight that is fluid:

A

60%

127
Q

Approximate percentage of body fluid that is extracellular:

A

33%

128
Q

Approximate percentage of body weight that is intracellular:

A

66%

129
Q

Percentage of extracellular fluid within the vascular compartment in the venous system:

A

85%

130
Q

Percentage of extracellular fluid within the vascular compartment in the arterial system:

A

15%

131
Q

The approximate percentage of body weight that blood accounts for in an adult:

A

7%

so to estimate how many liters of blood in a 70-kg man; 0.07 × 70 kg = 5 liters

132
Q

Requirement of water per 24-hour period:

A

~30 to 35 mL/kg

133
Q

Requirement of sodium per 24-hour period:

A

~1 to 2 mEq/kg

134
Q

Requirement of chloride per 24-hour period:

A

~1.5 mEq/kg

135
Q

Requirement of potassium per 24-hour period:

A

~1 mEq/kg

136
Q

Name the sources and the amount of normal daily water loss:

A

Respiratory losses: 500 to 700 cc
Sweat: 200 to 400 cc
Urine: 1200 to 1500 cc
Feces: 100 to 200 cc

137
Q

Name the sources and the amount of insensible fluid loss:

A

Skin: ~300 cc/24 h
Breathing: 500 to 700 cc/24 h
Feces: 100 to 200 cc/24 h

138
Q

General rate of fluid loss during an open abdominal procedure in the absence of measurable blood loss:

A

0.5 to 1.0 L/h

139
Q

Name the sources and the amount of normal daily electrolyte loss:

A

Chloride: 150 mEq
Sodium: 100 mEq
Potassium: 100 mEq

140
Q

Name the sources and the amount of daily secretions:

A
Saliva: ~1500 cc/24 h
Gastric: ~2000 cc/24 h
Small intestine: ~3000 cc/24 h
Bile: ~1000 cc/24 h 
Pancreatic: ~600 cc/24 h
141
Q

State the electrolyte composition of sweat:

A

30 to 50 mEq sodium
5 mEq potassium
45 to 55 mEq hydrogen

142
Q

State the electrolyte composition of gastric secretions:

A

40 to 65 mEq sodium
90 mEq hydrogen
100 to 140 mEq chloride

143
Q

State the electrolyte composition of biliary secretions:

A

135 to 155 mEq sodium
5 mEq potassium
80 to 110 mEq chloride
70 to 90 mEq bicarbonate

144
Q

State the electrolyte composition of pancreatic secretions:

A

135 to 155 mEq sodium
5 mEq potassium,
55 to 75 mEq chloride
70 to 90 mEq bicarbonate

145
Q

State the electrolyte composition of ileostomy output:

A

120 to 130 mEq sodium
10 mEq potassium
50 to 60 mEq chloride
50 to 70 mEq bicarbonate

146
Q

State the electrolyte composition of diarrhea:

A

25 to 50 mEq sodium
35 to 60 mEq potassium
20 to 40 mEq chloride
30 to 45 mEq bicarbonate

147
Q

Define the “third space.”

A

Fluid accumulation in the interstitium of tissues (first 2 spaces: intravascular and intracellular)

148
Q

When does third-spaced fluid tend to mobilize back into the intravascular space?

A

Postoperative day #3

149
Q

What is the earliest sign of volume excess during the postoperative period?

A

Weight gain

150
Q

Classic finding with overaggressive nasogastric tube suctioning or long-standing vomiting:

A

Hypokalemic hypochloremic metabolic alkalosis

151
Q

Name the various mechanisms that loop diuretics employ to decrease pulmonary edema:

A

Inhibit active sodium absorption in the thick ascending loop of Henle,
increase venous capacitance,
stimulate vasodilatory prostaglandins leading to increased renal blood flow

152
Q

Formula to calculate serum osmolality:

A

2 × sodium + urea/2.8 + glucose/18

153
Q

How much sodium and chloride are in normal saline?

A

154 mEq Na+

154 mEq Cl–

154
Q

Composition of lactated Ringer’s:

A
130 mEq Na+
109 mEq Cl–
4 mEq K+
28 mEq lactate
and 3 mEq calcium
155
Q

How many grams of dextrose in a liter of D5W?

A

50 g;
D5W is a 5% solution of dextrose
(5 g dextrose/100 cc × 1000 cc/1 L = 50 g dextrose)

156
Q

The 2 “rules” for the calculation of maintenance fluids:

A

100/50/20 rule and 4/2/1 rule; for both rules cc/kg for first 10 kg/cc/kg for next 10 kg/cc/kg for every kg >20 kg

157
Q

Name the standard maintenance fluid used in an adult:

A

D5 1/2 normal saline (NS) with 20 mEq KCl

158
Q

Name the standard maintenance fluid used in a pediatric patient:

A

D5 1/4 NS with 20 mEq KCl

159
Q

Usual minimal urine output for an adult:

A

~30 mL/h or 0.5 mL/kg/h

160
Q

How much of a 1-L NS bolus will stay intravascular in a 5-hour period?

A

~200 cc or 20%

161
Q

Normal range for sodium:

A

135 to 145 mEq/L

162
Q

Define pseudohyponatremia and list causes:

A

Spuriously low lab result for sodium; hyperglycemia,
hyperlipidemia,
hyperproteinemia

163
Q

Name the 3 types of hyponatremia:

A

Hypovolemic, euvolemic, hypervolemic

164
Q

Name surgical causes of hypovolemic hyponatremia:

A
Burns
diaphoresis
diuretics
hypoaldosteronism
NG suctioning
pancreatitis
vomiting
165
Q

Name surgical causes of euvolemic hyponatremia:

A

CNS abnormalities
drugs
syndrome of inappropriate secretion of antidiuretic hormone (SIADH)

166
Q

Name surgical causes of hypervolemic hyponatremia:

A

Congestive heart failure
cirrhosis
iatrogenic
renal failure

167
Q

Signs and symptoms of hyponatremia:

A
Coma/confusion
ileus
lethargy
nausea/vomiting
seizure
weakness
168
Q

Treatment of hypovolemic hyponatremia:

A

Correct the underlying cause, give IV NS

169
Q

Treatment of euvolemic hyponatremia (SIADH):

A

Acute treatment with furosemide and normal saline; fluid restriction

170
Q

Treatment of hypervolemic hyponatremia (dilutional):

A

Fluid restriction and diuretics

171
Q

Grave consequence of correcting hyponatremia too quickly:

A

Myelinolysis (formerly known as central pontine myelinolysis)

172
Q

Formula to calculate the sodium deficit:

A

(normal sodium concentration – observed sodium concentration) × total body water;

Remember: total body water = 0.6 × weight (kg)

173
Q

Approximately how much does the apparent serum sodium concentration fall for each 100 mg/dL rise in blood glucose level above normal?

A

1.6 to 3.0 mEq/ L

174
Q

Maximal rate of sodium correction for acute hyponatremia:

A

1 to 2 mEq/L/h for 3 to 4 hours until neurologic symptoms subside or
until plasma Na is >120 mEq/L

175
Q

Maximal rate of sodium correction for chronic hyponatremia:

A

0.5 to 1 mEq/L/h or no faster than 10 to 12 mEq/L in the first 24 hours and 18 mEq/L in the first 48 hours

176
Q

Name surgical causes of hypernatremia:

A
Dehydration, 
diabetes insipidus, 
diaphoresis, 
diarrhea, 
diuresis, 
iatrogenic, 
vomiting
177
Q

Signs and symptoms of hypernatremia:

A
Confusion, 
peripheral/pulmonary edema, respiratory paralysis, 
stupor, 
seizures, 
tremors
178
Q

Treatment of hypernatremia:

A

Slow supplementation of 1/4 NS or 1/2 NS over days

179
Q

Formula to calculate the free water deficit:

A

Total free water deficit =

0.6 × weight (kg) × [(Serum Na+/140) – 1]

180
Q

What is the normal range for potassium:

A

3.5 to 5.0 mEq/L

181
Q

Critical values for potassium:

A

K+ <2.8 mEq/L or >6.0 mEq/L

182
Q

Name some surgical causes of hypokalemia:

A
Alkalosis, 
diuretics, 
drugs (steroids/antibiotics), 
diarrhea, 
iatrogenic, 
insulin, 
intestinal fistula, 
NG suctioning, 
vomiting
183
Q

Signs and symptoms of hypokalemia:

A
Ileus, 
weakness, 
tetany, 
nausea/vomiting, 
paresthesia
184
Q

EKG findings of hypokalemia:

A
Flattened T waves, 
U waves, 
ST segment depression, 
atrial fibrillation, 
premature atrial complexes/
premature ventricular complexes
185
Q

Acute treatment for hypokalemia:

A

IV KCl

186
Q

Maximum amount of potassium that can be administered through a peripheral IV:

A

10 mEq/h

187
Q

Maximum amount of potassium that can be administered through a central line:

A

20 mEq/h

188
Q

Most common cause for an electrolyte mediated ileus in a surgical patient:

A

Hypokalemia

189
Q

Digitalis toxicity is worsened by this electrolyte condition:

A

Hypokalemia

190
Q

Name surgical causes of hyperkalemia:

A
Iatrogenic, 
diuretics, 
acidosis, 
trauma, 
hemolysis, 
renal failure, 
blood transfusion
191
Q

Signs and symptoms of hyperkalemia:

A
Areflexia or decreased deep tendon reflexes, 
paresthesia, 
paralysis, 
weakness, and 
respiratory failure
192
Q

EKG findings of hyperkalemia:

A
Peaked T waves, 
prolonged PR, 
wide QRS, 
depressed ST segment, 
ventricular fibrillation, 
bradycardia
193
Q

What is the treatment for hyperkalemia?

A
IV calcium, 
sodium bicarbonate, 
dextrose and 
insulin
 (1 amp 50% dextrose and 10 U insulin), 
albuterol, 
kayexalate, 
furosemide, 
dialysis
194
Q

Most of the calcium in the body is contained within:

A

Bone

195
Q

Percentage of serum calcium that is nonionized and bound to plasma protein:

A

~50%

196
Q

Percentage of serum calcium that is nonionized and bound to substances other than plasma protein in the plasma:

A

5%

197
Q

How much does a 1-g drop in protein decrease the measured total serum calcium?

A

0.8 mg/dL

198
Q

In the setting of rapid transfusion, how much calcium should be given per 500 cc of transfused blood?

A

0.2 g/500 cc transfused blood

199
Q

Name some surgical causes of hypocalcemia:

A
Acute pancreatitis, 
aminoglycosides, 
diuretics, 
hypomagnesemia, 
intestinal bypass, 
osteoblastic metastasis, 
renal failure, 
rhabdomyolysis, 
sepsis, 
short bowel syndrome, 
vitamin D deficiency
200
Q

Formula to determine the calcium level with hypoalbuminemia:

A

Serum calcium + [(4 – measured albumin) × 0.8]

201
Q

Signs and symptoms of hypocalcemia:

A
Abdominal cramping, 
Chvostek signs, 
confusion, 
depression, 
hallucinations, 
increased deep tendon reflexes, 
laryngospasm, 
paranoia, 
perioral paresthesia, 
seizures, 
stridor, 
tetany, 
Trousseau sign
202
Q

Define Chvostek sign:

A

Tapping of facial nerve with resultant facial muscle spasm

203
Q

Define Trousseau sign:

A

Utilization of a blood pressure cuff to occlude blood flow to the forearm with resultant latent tetany evidenced by carpal spasm

204
Q

EKG findings hypocalcemia:

A

Peaked T waves,

prolonged QT and ST intervals

205
Q

Acute treatment of hypocalcemia:

A

IV calcium supplementation

206
Q

Chronic treatment of hypocalcemia:

A

PO calcium and vitamin D supplementation

207
Q

Mnemonic for hypercalcemia:

A

Chimpanzees:

Calcium supplementation, 
Hyperparathyroidism (primary/tertiary)/hyperthyroidism, 
Iatrogenic/immobility, 
Milk alkali syndrome/mets, 
Paget disease, 
Addison disease/acromegaly, 
Neoplasm, 
Zollinger-Ellison syndrome, 
Excessive vitamin A, 
Excessive vitamin D, 
Sarcoid
208
Q

Signs and symptoms of hypercalcemia:

A

Polydipsia, polyuria, and constipation and the classic “stones, bones, abdominal groans, and psychiatric overtones”

209
Q

EKG findings of hypercalcemia:

A

Prolonged PR interval and

shortened QT interval

210
Q

Acute treatment for hypercalcemic crisis:

A

Normal saline volume expansion with furosemide diuresis

Additional options for treating hypercalcemia:
Bisphosphonates, 
calcitonin, 
mithramycin, 
steroids, and 
dialysis
211
Q

What kind of diuretic should be avoided in the face of hypercalcemia?

A

Thiazide diuretics (calcium-sparing diuretic)

212
Q

Normal range for magnesium:

A

1.5 to 2.5 mEq/L

213
Q

Electrolyte abnormality associated with “impossible to correct” hypokalemia and hypocalcemia:

A

Hypomagnesemia

214
Q

Name surgical causes of hypomagnesemia:

A

Aminoglycosides, diarrhea, gastric suctioning, hypocalcemia, renal failure, total parenteral nutrition (TPN), vomiting

215
Q

Signs and symptoms of hypomagnesemia:

A

Asterixis, Chvostek sign, dysrhythmias, increased deep tendon reflexes, tachycardia, tetany, tremor, ventricular ectopy, vertigo

216
Q

Treatment for acute hypomagnesemia:

A

IV magnesium sulfate (or magnesium chloride)

217
Q

Treatment for chronic hypomagnesemia:

A

PO magnesium oxide

218
Q

Name surgical causes of hypermagnesemia:

A

Iatrogenic, renal failure, TPN

219
Q

Signs and symptoms of hypermagnesemia:

A

IV calcium, dextrose and insulin, dialysis, Lasix

220
Q

Normal range for phosphorus:

A

2.5 to 4.5 mg/dL

221
Q

Name surgical causes of hypophosphatemia:

A

Alcohol abuse, gastrointestinal losses, inadequate supplementation, medications, renal loss, sepsis

222
Q

Critical value for phosphorus:

A

<1.0 mg/dL

223
Q

Signs and symptoms of hypophosphatemia:

A

Ataxia, cardiomyopathy, hemolysis, poor response to pressors, rhabdomyolysis, neurologic dysfunction, weakness

224
Q

Treatment for acute hypophosphatemia:

A

IV sodium phosphate or

potassium phosphate

225
Q

Treatment for chronic hypophosphatemia:

A

PO replacement (Neutra phos)

226
Q

Name surgical causes of hyperphosphatemia:

A

Chemotherapy, hyperthyroidism, renal failure, sepsis

227
Q

Signs and symptoms of hyperphosphatemia:

A

Calcification, heart block

228
Q

List the disturbances seen with tumor lysis syndrome:

A

Hypocalcemia,
hyperkalemia,
hyperuricemia,
hyperphosphatemia

229
Q

Name the important intracellular buffer:

A

Phosphate buffer system and proteins

230
Q

Name the primary extracellular buffering system:

A

Bicarbonate-carbonic acid system

231
Q

Henderson-Hasselbalch equation:

A

pH = pK + log [HCO–3]/[CO2]

232
Q

Formula to calculate the anion gap:

A

Anion gap = Sodium – (bicarbonate + chloride)

233
Q

Mnemonic for anion gap acidosis:

A
Mudpiles: 
methanol, 
uremia, 
diabetic ketoacidosis, 
paraldehyde, 
isoniazid, 
lactic acidosis, 
ethylene glycol, 
salicylates
234
Q

Formula for functional excretion of sodium (FeNa):

A

FeNa = (urine sodium/urine creatinine)/(plasma sodium/plasma creatinine)

235
Q

List some common laboratory values seen in a prerenal state:

A

BUN/Cr ratio >20,
FeNa <1%,
urine sodium <20,
urine osmolality >500 mOsm

236
Q

How does relative glucose intolerance present?

A

Manifests as glycosuria.

This may occur after initiation of parenteral nutrition. If blood glucose remain elevated or glycosuria persists, dextrose concentration may be decreased, infusion rate slowed, or regular insulin added to each bottle.

The rise in blood glucose may be temporary as the normal pancreas increases its output of insulin in response to the continuous carbohydrate infusion.